You are on page 1of 92

Japan

Todays Calculation Of Integral


2005
1 Calculate the following indenite integral.
[1]
_
e
2x
(e
x
+1)
2
dx
[2]
_
sin xcos 3xdx
[3]
_
sin 2xsin 3xdx
[4]
_
dx
4x
2
12x+9
[5]
_
cos
4
xdx
2 Calculate the following indenite integrals.
[1]
_
cos
_
2x

3
_
dx
[2]
_
dx
cos
2
(3x+4)
[3]
_
(x 1)
3

x 2dx
[4]
_
x 3
x
2
+1
dx
[5]
_
dx

1x
dx
3 Calculate the following indenite integrals.
[1]
_
sin xsin 2xdx
[2]
_
e
2x
e
x
1
dx
[3]
_
tan
2
x
cos
2
x
dx
[4]
_
e
x
+e
x
e
x
e
x
dx
[5]
_
e
x
e
x
+1
dx
4 Calculate the following indenite integrals.
[1]
_
x

5x
dx
[2]
_
sin xcos
2
x
1+cos x
dx
[3]
_
(sin x + cos x)
2
dx
[4]
_
xcos
2
x
xcos
2
x
dx
[5]
_
(sin x + sin 2x)
2
dx
5 Calculate the following indenite integrals.
[1]
_
(4 5 tan x) cos xdx
[2]
_
dx
3

(13x)
2
dx
http://www.artofproblemsolving.com/
This le was downloaded from the AoPS Math Olympiad Resources Page Page 1

Japan
Todays Calculation Of Integral
2005
[3]
_
x
3

4 x
2
dx
[4]
_
e
x
sin
_
x +

4
_
dx
[5]
_
(3x 4)
2
dx
6 Calculate the following indenite integrals.
[1]
_
sin xcos
3
xdx
[2]
_
dx
(1+

x)

x
dx
[3]
_
x
2

x
3
+ 1dx
[4]
_
e
2x
3e
x
e
x
dx
[5]
_
(1 x
2
)e
x
dx
7 Calculate the following indenite integrals.
[1]
_
x(

x + 1)
2
dx
[2]
_
(e
x
+ 2e
x+1
3e
x+2
)dx
[3]
_
(sin
2
x + cos x) sin xdx
[4]
_
x

2 xdx
[5]
_
xln xdx
8 Calculate the following indenite integrals.
[1]
_
x(x
2
+ 3)
2
dx
[2]
_
ln(x + 2)dx
[3]
_
xcos xdx
[4]
_
dx
(x+2)
2
dx
[5]
_
x1
x
2
2x+3
dx
9 Calculate the following indenite integrals.
[1]
_
(x
2
+ 4x 3)
2
(x + 2)dx
[2]
_
ln x
x(ln x+1)
dx
[3]
_
sin ( log
2
x)
x
dx
[4]
_
dx
sin xcos
2
x
[5]
_
1 3x dx
http://www.artofproblemsolving.com/
This le was downloaded from the AoPS Math Olympiad Resources Page Page 2

Japan
Todays Calculation Of Integral
2005
10 Calculate the following indenite integrals.
[1]
_
(2x + 1)

x + 2 dx
[2]
_
1+cos x
x+sin x
dx
[3]
_
sin
5
xcos
3
x dx
[4]
_
(x3)
2
x
4
dx
[5]
_
dx
tan x
dx
11 Calculate the following indenite integrals.
[1]
_
6x+1

3x
2
+x+4
dx
[2]
_
e
x
e
x
+e
ax
dx
[3]
_
(

x+1)
3

x
dx
[4]
_
xln(x
2
1)dx
[5]
_
2(x+2)
x
2
+4x+1
dx
12 Calculate the following indenite integrals.
[1]
_
dx
1+cos x
[2]
_
x

x
2
1dx
[3]
_
a

x
2
dx (a > 0, a = 1)
[4]
_
sin
3
x
1+cos x
dx
[5]
_
e
4x
sin 2xdx
13 Calculate the following integarls.
[1]
_
xcos
2
xdx
[2]
_
x1
(3x1)
2
dx
[3]
_
x
3
(2x
2
)
4
dx
[4]
_
_
1
4

x
+
1
2x
_
dx
[5]
_
(ln x)
2
dx
14 Calculate the following indenite integrals.
[1]
_
sin xcos x
1+sin
2
x
dx
[2]
_
xlog
10
xdx
http://www.artofproblemsolving.com/
This le was downloaded from the AoPS Math Olympiad Resources Page Page 3

Japan
Todays Calculation Of Integral
2005
[3]
_
x

2x1
dx
[4]
_
(x
2
+ 1) ln xdx
[5]
_
e
x
cos xdx
15 Calculate the following indenite integrals.
[1]
_
(x
2
1)
2
x
4
dx
[2]
_
e
3x

e
x
+1
dx
[3]
_
sin 2xcos 3xdx
[4]
_
xln(x + 1)dx
[5]
_
x
(x+3)
2
dx
16 Calculate the following indenite integrals.
[1]
_
sin(ln x)dx
[2]
_
x+sin
2
x
xsin
2
x
dx
[3]
_
x
3
x
2
+1
dx
[4]
_
x
2

2x1
dx
[5]
_
x+cos 2x+1
xcos
2
x
dx
17 Calculate the following indenite integrals.
[1]
_
dx
e
x
e
x
[2]
_
e
ax
cos 2xdx (a = 0)
[3]
_
(3
x
2)
2
dx
[4]
_
x
4
+2x
3
+3x
2
+1
(x+2)
5
dx
[5]
_
dx
1cos x
dx
18 Calculate the following indenite integrals.
[1]
_
(sin x + cos x)
4
dx
[2]
_
e
2x
e
x
+1
dx
[3]
_
sin
4
xdx
[4]
_
sin 6xcos 2xdx
[5]
_
x
2

(x+1)
3
dx
http://www.artofproblemsolving.com/
This le was downloaded from the AoPS Math Olympiad Resources Page Page 4

Japan
Todays Calculation Of Integral
2005
19 Calculate the following indenite integrals.
[1]
_
tan
3
xdx
[2]
_
a
mx+n
dx (a > 0, a = 1, mn = 0)
[3]
_
cos
5
xdx
[4]
_
sin
2
xcos
3
xdx
[5]
_
dx
sin x
20 Calculate the following indenite integrals.
[1]
_
ln(x
2
1)dx
[2]
_
1
e
x
+1
dx
[3]
_
(ax
2
+ bx + c)e
mx
dx (abcm = 0)
[4]
_ _
tan x +
1
tan x
_
2
dx
[5]
_
1 sin xdx
21 [1] Tokyo Univ. of Science:
_
ln x
(x+1)
2
dx
[2] Saitama Univ.:
_
5
3 sin x+4 cos x
dx
[3] Yokohama City Univ.:
_

3
1
1

x
2
+1
dx
[4] Daido Institute of Technology:
_

2
0
sin
3
x
sin x+cos x
dx
[5] Gunma Univ.:
_
3
4
0
{(1 + x) sin x + (1 x) cos x}dx
22 Evaluate
_
1
0
(1 x
2
)
n
dx (n = 0, 1, 2, )
23 Evaluate
lim
a

2
0
_
a
0
(cos x) ln(cos x) dx
_
0 a <

2
_
24 Find the minimum value of
_

0
(x y)
2
(sin x)| cos x|dx.
http://www.artofproblemsolving.com/
This le was downloaded from the AoPS Math Olympiad Resources Page Page 5

Japan
Todays Calculation Of Integral
2005
25 Let |a| <

2
.
Evaluate
_
2
0
dx
{sin(a + x) + cos x}
2
26 Evaluate
_
e
e
e
e
e
e
e
dx
xln x ln(ln x) ln{ln(ln x)}
27 Let f(x) = t sin x + (1 t) cos x (0 t 1).
Find the maximum and minimum value of the following P(t).
P(t) =
_
_
2
0
e
x
f(x)dx
__
_
2
0
e
x
f(x)dx
_
28 Evaluate
_
4
0
xcos 5x
cos x
dx
29 Let a be a real number. Evaluate
_
3+a
+a
|x a | sin
_
x
2
_
dx
30 A sequence {a
n
} is dened by a
n
=
_
1
0
x
3
(1 x)
n
dx (n = 1, 2, 3. )
Find the constant number c such that

n=1
(n + c)(a
n
a
n+1
) =
1
3
31 Evaluate
lim
n
_

0
x
2
| sin nx|dx
32 Evaluate
_
1
0
e
x+e
x
+e
e
x
+e
e
e
x
dx
http://www.artofproblemsolving.com/
This le was downloaded from the AoPS Math Olympiad Resources Page Page 6

Japan
Todays Calculation Of Integral
2005
33 Evaluate
_
0
ln 2
dx
cos
2
hx

1 2a tanh x + a
2
(a > 0)
34 Let p be a constant number such that 0 < p < 1. Evaluate
2004

k=0
p
k
(1 p)
2004k
_
1
0
x
k
(1 x)
2004k
dx
35 Determine the value of a, b for which
_
1
0
(

1 x ax b)
2
dx is minimized.
36 A sequence of polynomial f
n
(x) (n = 0, 1, 2, ) satises f
0
(x) = 2, f
1
(x) = x,
f
n
(x) = xf
n1
(x) f
n2
(x), (n = 2, 3, 4, )
Let x
n
(n 2) be the maximum real root of the equation f
n
(x) = 0 (|x| 2)
Evaluate
lim
n
n
2
_
2
xn
f
n
(x)dx
37 Evaluate
_ 2
3

2
1
sin x

1 cos x
dx
38 Let a be a constant number such that 0 < a < 1 and V (a) be the volume formed by the
revolution of the gure
which is enclosed by the curve y = ln(x a), the x-axis and two lines x = 1, x = 3 about the
x-axis.
If a varies in the range of 0 < a < 1, nd the minimum value of V (a).
39 Find the minimum value of the following function f(x) dened at 0 < x <

2
.
f(x) =
_
x
0
d
cos
+
_
2
x
d
sin
http://www.artofproblemsolving.com/
This le was downloaded from the AoPS Math Olympiad Resources Page Page 7

Japan
Todays Calculation Of Integral
2005
40 Evaluate
_
1
0
x
2005
e
x
2
dx
41 Evaluate
_
a
0
_
2ax x
2
dx (a > 0)
42 Let 0 < t <

2
.
Evaluate
lim
t

2
_
t
0
tan

cos ln(cos )d
43 Evaluate
_
2
0
cos
2004
xcos 2004x dx
44 Evaluate
_
2
0
sin 2005x
sin x
dx
45 Find the function f(x) which satises the following integral equation.
f(x) =
_
x
0
t(sin t cos t)dt +
_
2
0
e
t
f(t)dt
46 Find the minimum value of
_
1
0
|tx|
t+1
dt
47 Find the condition of a, b for which the function f(x) (0 x 2) satisfying the following
equality can be determined uniquely,then determine f(x), assuming that f(x) is a continuous
function at 0 x 2.
f(x) =
a
2
_
2
0
sin(x + y)f(y)dy +
b
2
_
2
0
cos(x y)f(y)dy + sin x + cos x
http://www.artofproblemsolving.com/
This le was downloaded from the AoPS Math Olympiad Resources Page Page 8

Japan
Todays Calculation Of Integral
2005
48 Evaluate
lim
n
_
_

0
sin
2
nx
sin x
dx
n

k=1
1
k
_
49 For x 0, Prove that
_
x
0
(t t
2
) sin
2002
t <
1
20042005
50 Let a, b be real numbers such that a < b. Evaluate
lim
ba
_
b
a
ln |1 + (x a)(b x)|dx
(b a)
3
.
51 A function f(x) satises f(x) = f
_
c
x
_
for some real number c(> 1) and all positive number
x.
If
_

c
1
f(x)
x
dx = 3, evaluate
_
c
1
f(x)
x
dx
52 Evaluate
lim
n
n

k=1
1
n + k

1
53 Find the maximum value of the following integral.
_

0
e
x
sin tx dx
54 evaluate
_
0
1
_
1 + x
1 x
dx
55 Evaluate
lim
n
n
_
1
0
(1 + x)
n1
e
x
2
dx (n = 1, 2, )
http://www.artofproblemsolving.com/
This le was downloaded from the AoPS Math Olympiad Resources Page Page 9

Japan
Todays Calculation Of Integral
2005
56 Evaluate
lim
n
n

k=1
[

2n
2
k
2
]
n
2
[x] is the greatest integer x.
57 Find the value of n N satisfying the following inequality.

_

0
x
2
sin nx dx

<
99
2
100n
58 Let f(x) =
e
x
e
x
+1
Prove the following equation.
_
b
a
f(x)dx +
_
f(b)
f(a)
f
1
(x)dx = bf(b) af(a)
59 Evaluate
_

(cos 2x)(cos 2
2
x) (cos 2
2006
x)dx
60 Let a
n
=
_

2
0
sin 2t (1 sin t)
n1
2
dt (n = 1, 2, )
Evaluate

n=1
(n + 1)(a
n
a
n+1
)
61 Evaluate
2004

k=0
_
1
1

1 x
2

k + 1 x
dx
62 For a > 1, let f(a) =
1
2
_
1
0
|ax
n
1|dx +
1
2
(n = 1, 2, ) and let b
n
be the minimum value of
f(a) at a > 1.
Evaluate
http://www.artofproblemsolving.com/
This le was downloaded from the AoPS Math Olympiad Resources Page Page 10

Japan
Todays Calculation Of Integral
2005
lim
m
b
m
b
m+1
b
2m
(m = 1, 2, 3, )
63 For a positive number x, let f(x) = lim
n

n
k=1

cos
_
2k+1
2n
x
_
cos
_
2k1
2n
x
_

Evaluate
lim
x
f(x)
x
64 Let f(t) be the cubic polynomial for t such that cos 3x = f(cos x) holds for all real number
x. Evaluate
_
1
0
{f(t)}
2
_
1 t
2
dt
65 Let a > 0. Find the minimum value of
_
a
1
_
1
x
a
_
1 + x dx
66 Find the minimum value of
_

2
0
| cos x a| sin x dx
67 Evaluate
2005
_
1002
0
dx

1002
2
x
2
+

1003
2
x
2
+
_
1003
1002
_
1003
2
x
2
dx
_
1
0
_
1 x
2
dx
68 Find the minimum value of
_
e
1

ln x
a
x

dx (0 a e)
69 Let f
1
(x) = x, f
n
(x) = x +
1
14
_

0
xf
n1
(t) cos
3
t dt (n 2).
Find lim
n
f
n
(x)
70 Find the number of root for
_

2
0
e
x
cos(x + a) dx = 0 at 0 a < 2
71 Find the minimum value of
_
1
1
_
|t x| dt
72 Let f(x) be a continuous function satisfying f(x) = 1+k
_

2

2
f(t) sin(xt)dt (k : constant number)
Find the value of k for which
_

0
f(x)dx is maximized.
73 Find the minimum value of
_

0
(a sin x + b sin 2x + c sin 3x x)
2
dx
http://www.artofproblemsolving.com/
This le was downloaded from the AoPS Math Olympiad Resources Page Page 11

Japan
Todays Calculation Of Integral
2005
74 p, q satises px + q ln x at a x b (0 < a < b).
Find the value of p, q for which the following denite integral is minimized and then the
minimum value.
_
b
a
(px + q ln x)dx
75 A function f() satises the following conditions (a), (b).
(a) f() 0
(b)
_

0
f() sin d = 1
Prove the following inequality.
_

0
f() sin n d n (n = 1, 2, )
76 The function f
n
(x) (n = 1, 2, ) is dened as follows.
f
1
(x) = x, f
n+1
(x) = 2x
n+1
x
n
+
1
2
_
1
0
f
n
(t) dt (n = 1, 2, )
Evaluate
lim
n
f
n
_
1 +
1
2n
_
77 Find the area of the part enclosed by the following curve.
x
2
+ 2axy + y
2
= 1 (1 < a < 1)
78 Let , be the distinct positive roots of the equation of 2x = tan x. Evaluate
_
1
0
sin xsin x dx
79 Find the area of the domain expressed by the following system inequalities.
x 0, y 0, x
1
p
+ y
1
p
1 (p = 1, 2, )
http://www.artofproblemsolving.com/
This le was downloaded from the AoPS Math Olympiad Resources Page Page 12

Japan
Todays Calculation Of Integral
2005
80 Let S be the domain surrounded by the two curves C
1
: y = ax
2
, C
2
: y = ax
2
+ 2abx for
constant positive numbers a, b. Let V
x
be the volume of the solid formed by the revolution of
S about the axis of x, V
y
be the volume of the solid formed by the revolution of S
about the axis of y. Find the ratio of
Vx
Vy
.
81 Prove the following inequality.
1
12
( 6 + 2

3)
_
4

6
ln(1 + cos 2x)dx
1
4
(2

3)
82 Let 0 < a < b.Prove the following inequaliy.
1
b a
_
b
a
_
ln
b
x
_
2
dx < 2
83 Evaluate

n=1
_
2(n+1)
2n
xsin x + cos x
x
2
dx (n = 1, 2, )
84 Evaluate
lim
n
n
_

0
e
nx
sin
2
nx dx
85 Evaluate
lim
n
_
2
0
[nsin x]
n
dx
where [x] is the integer equal to x or less than x.
86 Prove
__

cos x
x
dx
_
2
<
1

2
http://www.artofproblemsolving.com/
This le was downloaded from the AoPS Math Olympiad Resources Page Page 13

Japan
Todays Calculation Of Integral
2005
87 Find the minimum value of a (0 < a < 1) for which the following denite integral is minimized.
_

0
| sin x ax| dx
88 A function f(x) satises
_
f(x) = f

(x) (4x 2)f

(x)
f(0) = a, f(1) = b
Evaluate
_
1
0
f(x)(x
2
x) dx.
89 For f(x) = x
4
+|x|, let I
1
=
_

0
f(cos x) dx, I
2
=
_

2
0
f(sin x) dx.
Find the value of
I
1
I
2
.
90 Find lim
n
_
3n
Cn
2n
Cn
_1
n
where
i
C
j
is a binominal coecient which means
i(i1)(ij+1)
j(j1)21
.
91 Prove the following inequality.

n=0
_
1
0
x
4011
(1 x
2006
)
n1
2006
dx <
2006
2005
http://www.artofproblemsolving.com/
This le was downloaded from the AoPS Math Olympiad Resources Page Page 14

Japan
Todays Calculation Of Integral
2006
92 Evaluate lim
n
n
2
_
1
n

1
n
(2005 sin x + 2006 cos x)|x|dx.
93 Evaluate
1
_
2
0
cos
2005
x sin 2007x dx
.
94 Let a be real numbers.Find the following limit value.
lim
T
1
T
_
T
0
(sin x + sin ax)
2
dx.
95 Evaluate
_
3

4
2 sin
3
x
cos
5
x
dx.
96 For a 0, nd the minimum value of
_
1
2
|x
2
+ 2ax|dx.
97 Answer the following questions.
(1) Evaluate
_
e
e
e
ln(ln x)
xln x
dx.
(2) Let , be real numbers.Find the values of , for which the following equality holds for
any real numbers p, q.
_
2

2
(p cos x + q sin x)(x
2
+ x + )dx = 0.
98 Let
I
n
=
_
1+
1
n
1
{[(x + 1) ln x + 1]e
x(e
x
ln x+1)
+ n}dx (n = 1, 2, ).
Evaluate lim
n
I
n
n
.
99 Let be a constant number such that 0 . Evaluate
_
2
0
sin 8x| sin(x )| dx.
http://www.artofproblemsolving.com/
This le was downloaded from the AoPS Math Olympiad Resources Page Page 1

Japan
Todays Calculation Of Integral
2006
100 Let a, b, c be positive numbers such that abc =
1
16
. Prove the following inequality.
_

0
x
2
(x
2
+ a
2
)(x
2
+ b
2
)(x
2
+ c
2
)
dx .
101 Thank you very much, vidyamanohar. I will continue to post problems.
For n > 2, prove the following inequality.
1
2
<
_ 1
2
0
1

1 x
n
dx <

6
.
102 Let a, b be costant numbers such that a
2
b. Find the following indenite integrals.
(1) I =
_
dx
x
2
+2ax+b
(2) J =
_
dx
(x
2
+2ax+b)
2
103 For 0 < a < 1, let f(x) =
ax
1ax
(1 < x < 1).
Evaluate
_
a
0
1{f(x)}
6
1x
2
dx.
104 For 0 < x < 1, let f(x) =
_
x
0
dt

1t
2
dt
(1) Find
d
dx
f(

1 x
2
)
(2) Find f
_
1

2
_
(3) Prove that f(x) + f(

1 x
2
) =

2
105 Let a, b be constant numbers such that 0 < a < b. If a function f(x) always satises f

(x) > 0
at a < x < b, for a < t < b nd the value of t for which the following the integral is minimized.
_
b
a
|f(x) f(t)|x dx.
106 Evaluate
_
1
0
1x
2
1+x
2
dx

1+x
4
107 Evaluate
_
1
1

1x
2
ax
dx (a > 1)
108 For x 0, nd the minimum value of x for which
_
x
0
2
t
(2
t
3)(x t) dt is minimized.
109 Let I
n
=
_
2006+
1
n
2006
xcos
2
(x 2006) dx (n = 1, 2, ).
Find lim
n
nI
n
.
http://www.artofproblemsolving.com/
This le was downloaded from the AoPS Math Olympiad Resources Page Page 2

Japan
Todays Calculation Of Integral
2006
110 Prove the following inequality.
1
_
2
0
_
1 sin
3
x dx
1
2
{

2 + ln(1 +

2 )}
111 Evaluate
_

2

2
e
nx
cos
m
x dx (m, n = 0, 1, 2, ).
112 Evaluate
_

2
0
d
(1e
2
sin
2
)
3
(e < 1 is a constant number).
113 Evaluate
_

3

sin x+

cos x+3(

sin x

cos x) cos 2x

sin 2x
dx.
114 Let a be positive numbers.For |x| a, nd the maximum and minimum value of
_
x+a
xa

4a
2
t
2
dt.
115 Find the value of a such that
_

2
0
(sin x + a cos x)
3
dx
4a
2
_

2
0
xcos xdx = 2.
116 Find lim
t0
_
2
0
| sin(x+t)sin x| dx
|t|
.
117 Let a be a real constant number. Evaluate lim
n
n
_
1
1
e
n|ax|
dx.
118 Let f(x) be the function dened for x 0 which satises the following conditions. (a)
f(x) =
_
x (0 x < 1)
2 x (1 x < 2)
(b) f(x + 2n) = f(x) (n = 1, 2, )
Find lim
n
_
2n
0
f(x)e
x
dx.
119 Find the continuous function f(x) and constant number such that
_
x
0
f(t) dt = e
x
ae
2x
_
1
0
f(t)e
t
dt.
120 Let k be real constants.How many real roots can the following quadratic equation have?
x
2
= 2x + k +
_
1
0
|t + k| dt.
121 Given the parabola C : y = x
2
. If the circle centered at y axis with radius 1 has common
tangent lines with C at distinct two points, then nd the coordinate of the center of the circle
K and the area of the gure surrounded by C and K.
122 Let x(t) = tan t, y(t) = ln cos t
_

2
< t <

2
_
. Find the area surrounded by the curve
: x = x(t), y = y(t) and x axis and the line x = 1.
http://www.artofproblemsolving.com/
This le was downloaded from the AoPS Math Olympiad Resources Page Page 3

Japan
Todays Calculation Of Integral
2006
123 Let f(x) = x
2
sin x
2
. Prove that the volume V formed by the revolution of the gure
surrounded by the part 0 x 1 of the graph of y = f(x) and x axis about y axis is can be
given as 2
_
1
0
xf(x) dx then nd the value of V.
124 Let a > 1. Find the area S(a) of the part surrounded by the curve y =
a
4

(a
2
x
2
)
3
(0 x
1), x axis , y axis and the line x = 1, then when a varies in the range of a > 1, then nd the
extremal value of S(a).
125 Prove the following inequality for x 0.
_
x
0
(t t
2
) sin
2004
t dt <
1
2006
126 For t > 0, nd the minimum value of
_
1
0
x|e
x
2
t|dx.
127 Evaluate
_
e

3
e

4
1
sin(2 ln x)
dx.
This problem is mistaken
128 Prove the following inequality.

3
ln 2 +

3
81
<
_
3
0
ln(cos x)dx <

3
162
.
129 The sequence {a
n
} is dened as follows.
a
1
=

4
, a
n
=
_ 1
2
0
(cos x + a
n1
) cos x dx (n = 2, 3, )
Find lim
n
a
n
.
130 Find the value of a such that
_
a
0
1
e
x
+4e
x
+5
dx = ln
3

2.
131 For a > 0, nd the minimum value of
_
1
a
0
(a
3
+ 4x a
5
x
2
)e
ax
dx.
132 Find the area of the gure such that the points (x, y) satises the inequality lim
n
(x
2n
+
y
2n
)
1
n

3
2
x
2
+
3
2
y
2
1.
133 Let f(x) be the polynomial with respect to x, and g
n
(x) = n2n
2

x
1
2

n 2n
2

x
1
2

.
Find lim
n
_
1
0
f(x)g
n
(x) dx.
134 For positive integers n, let A
n
=
1
n
{(n + 1) + (n + 2) + + (n + n)}, B
n
= {(n + 1)(n +
2) (n + n)}
1
n
. Find lim
n
An
Bn
.
http://www.artofproblemsolving.com/
This le was downloaded from the AoPS Math Olympiad Resources Page Page 4

Japan
Todays Calculation Of Integral
2006
135 Find the value of a for which
_

2
0
|a sin x cos x| dx (a > 0) is minimized.
136 Let c be the constant number such that c > 1.Find the least area of the gure surrounded by
the line passing through the point (1, c) and the palabola y = x
2
on x y plane.
137 Find the value of a for which
_
1
0
|xe
x
a| dx is minimized.
138 Let f(x) be the product of functions made by taking four functions from three functions
x, sin x, cos x repeatedly. Find the minimum value of
_

2
0
f(x) dx.
139 Let a, b be real numbers. Evaluate
_
2
0
(a cos x + b sin x)
2n
dx (n = 1, 2, ).
140 Evaluate
_
4
0
_
cos x
sin x + cos x
_
2
dx,
_
4
0
_
sin x + cos x
cos x
_
2
dx.
141 Evaluate
_

0
cos 4xcos 4
cos xcos
dx.
142 Evaluate
_

0
sin x

12a cos x+a


2
dx (a > 0).
143 Evaluate
_

2
0
1sin 2x
(1+sin 2x)
2
dx.
144 Evaluate lim
n
_

_
x +

n
_
sin nx

dx (n = 1, 2, ).
145 Find the minimum value of
_
x+l
x
_
t +
1
t
_
dt (x > 0, l > 0).
146 Find the maximum value of
_
1
1
|x a|e
x
dx for |a| 1.
147 Find the area of the gure surrounded by the curve 2(x
2
+ 1)y
2
+ 8x
2
y + x
4
+ 4x
2
1 = 0.
148 Evaluate
_

2
0
sin
2
n
sin
2

d (n = 1, 2, ).
149 Let f(x) = (1 x
2
)
3
2
. Denote M the maximum value of |f

(x)| in (1, 1). Prove that


_
1
1
f(x) dx M.
1981 Musashi Institute of Technology entrance exam/Electro Telecommunication, Civil
Enginerring
150 Find the value of a such that lim
n
3
2
_ 3

a
_
1
t
3
n
_
n
t
2
dt =

2 (n = 1, 2, ).
http://www.artofproblemsolving.com/
This le was downloaded from the AoPS Math Olympiad Resources Page Page 5

Japan
Todays Calculation Of Integral
2006
151 Let a, b be positive constant numbers.Find the volume of the revolution of the region sur-
rounded by the parabola y = ax
2
and the line y = bx about the line y = bx as the axis on xy
plane.
152 Let f(x) the function such that f(0) = 0, |f

(x)|
1
1+x
(x 0). Prove that
_
e1
0
{f(x)}
2
dx
e 2.
153 Draw the perpendicular to the tangent line of the ellipse
x
2
a
2
+
y
2
b
2
= 1 (a > 0, b > 0) from the
origin O(0, 0).
Let be the angle between the perpendicular and the positive direction of x axis. Denote
the length of the perpendicular by r().
Calculate
_
2
0
r()
2
d.
154 Find the function f(x) which is dened for
_

2
,

2

such that f(x)+


_

2

2
sin(xy)f(y) dy =
x + 1
_

2
x

2
_
.
155 The sequence {c
n
} is determined by the following equation.
c
n
= (n + 1)
_
1
0
x
n
cos x dx (n = 1, 2, ).
Let be the limit value lim
n
c
n
. Find lim
n
c
n+1

cn
.
156 For arbiterary integers n, nd the continuous function f(x) which satises the following
equation.
lim
h0
1
h
_
x+nh
xnh
f(t)dt = 2f(nx).
Note that x can range all real numbers and f(1) = 1.
157 Find the volume of the solid expressed by the following six inequaities in xyz space.
x 0, y 0, z 0, x + y + z 3, x + 2z 4, y z 1.
158 (1) Evaluate the denite integral
_

0
e
x
sin xdx.
(2) Find the limit lim
n
_
n
0
e
x
| sin x|dx.
159 A function is dened by f(x) =
_
x
0
1
1+t
2
dt.
(1) Find the equation of normal line at x = 1 of y = f(x).
(2) Find the area of the gure surrounded by the normal line found in (1), the x axis and the
graph of y = f(x).
Note that you may not use the formula
_
1
1+x
2
dx = tan
1
x + Const.
http://www.artofproblemsolving.com/
This le was downloaded from the AoPS Math Olympiad Resources Page Page 6

Japan
Todays Calculation Of Integral
2006
160 Find the value of m (0 < m < 1) for which
_

0
| sin x mx| dx is minimized.
161 Find the dierentiable function f(x) such that f(x) =
_
x
0
f(t) tan t dt+
_
x
0
tan(tx) dt
_
|x| <

2
_
.
162 Let f(x) be the function such that f(x) > 0 at x 0 and {f(x)}
2006
=
_
x
0
f(t)dt + 1.
Find the value of {f(2006)}
2005
.
163 Let I
n
=
_

4
0
tan
n
x dx (n = 0, 1, 2, ).
Find

n=0
{I
n+2
2
+ (I
n+1
+ I
n+3
)I
n+2
+ I
n+1
I
n+3
}.
164 For positive integers n, let
S
n
=
1

1
+
1

2
+ +
1

n
, T
n
=
1

1+
1
2
+
1

2+
1
2
+ +
1

n+
1
2
.
Find lim
n
Tn
Sn
.
165 On xy plane, let C : y = 2006x
3
12070102x
2
+ . Find the area of the region surrounded
by the tangent line of C at x = 2006 and the curve C.
166 Express the following the limit values in terms of a denite integral and nd them.
(1) I = lim
n
1
n
ln
_
1 +
1
n
_ _
1 +
2
n
_

_
1 +
n
n
_
.
(2) J = lim
n
1
n
2
(

n
2
1 +

n
2
2
2
+ +

n
2
n
2
).
(3) K = lim
n
1
n
3
(

n
2
+ 1 + 2

n
2
+ 2
2
+ + n

n
2
+ n
2
).
167 In xyz plane nd the volume of the solid formed by the points (x, y, z) satisfying the following
system of inequalities.
0 z 1 + x + y 3(x y)y, 0 y 1, y x y + 1.
http://www.artofproblemsolving.com/
This le was downloaded from the AoPS Math Olympiad Resources Page Page 7

Japan
Todays Calculation Of Integral
2008
257 Evaluate
_

_
2008
0
x| sin x| dx
258 Find the volume of the solid formed by the revolution of the curve x = a(1+ cos ) cos , y =
a(1 + cos ) sin (a > 0, < ) about the x axis.
259 Evaluate
_

12
0
dx
(sin x+cos x)
4
260 Evaluate
_

3

4
(sin
3
cos
3
cos
2
)(sin +cos +cos
2
)
2007
sin
2009
cos
2009

d
261 Find the continuous function f(x) such that f(x) =
_
x
0
{f(t) cos t cos(t x)} dt.
262 Answer the following questions for positive integer n.
(1) Find the maximum value of f
n
(x) = x
n
e
x
for x 0.
(2) Show that lim
x
f
n
(x) = 0.
(3) Let I
n
(x) =
_
x
0
f
n
(t) dt, nd lim
x
I
n
(x).
263 Let F(t) =
1
t
_

2
t
0
| cos 2x| dx for 0 < t 1.
(1) Find lim
t0
F(t).
(2) Find the range of t such that F(t) 1.
264 Find the area of the gure surrounded by the locus of the point P inside the square with side
length a such that the distance to the center of the square is equal to the minimum distance
to the side of the square.
265 Supposed that f(x) has f

(x) and for any real numbers x, y,


_
x+y
y
f(t) dt =
_
x
0
{f(y) cos t +
f(t) cos y} dt holds.
(1) Express f(x + y) in terms of f(x), f(y). (2) Find f

_
x +

2
_
. Note that f
_

2
_
= 2. (3)
Find f(x).
266 Find the area of the region expressed by the system of inequality
x
2
4
+y
2
1,
_
1 +

3
2
_
x+y
1.
267 Let K be the curved surface obtained by rotating the parabola y =
3
4
x
2
about the y-
axis.Cut K by the plane H passing through the origin and forming angle 45

for the axis.


Find the volume of the solid surrounded by K and H.
Note that you are not allowed to use Double Integral for the problem.
268 Find the constant numbers u, v, s, t (s < t) such that
_
1
1
f(x) dx = uf(s) +vf(t) holds for
any polynomials f(x) = ax
3
+bx
2
+cx +d with the degree of 3 or less than.
http://www.artofproblemsolving.com/
This le was downloaded from the AoPS Math Olympiad Resources Page Page 1

Japan
Todays Calculation Of Integral
2008
269 For the curve C : y =
1
1+x
2
, Let A(, f()), B
_

, f
_

__
( > 0). Find the minimum
area bounded by the line segments OA, OB and C, where O is the origin.
Note that you are not allowed to use the integral formula of
1
1+x
2
for the problem.
270 Let f(x) be the continuous function at 0 x 1 such that
_
1
0
x
k
f(x) dx = 0 for integers
k = 0, 1, , n 1 (n 1).
(1) For all real numbers t, nd the minimum value of g(t) =
_
1
0
|x t|
n
dx.
(2) Show the following equation for all real real numbers t.
_
1
0
(x t)
n
f(x) dx =
_
1
0
x
n
f(x) dx
(3) Let M be the maximum value of the function |f(x)| for 0 x 1.
Show that

_
1
0
x
n
f(x) dx


M
2
n
(n+1)
271 For a positive constant number a, the function f(x) satises the following equation.
f(x) = xe

x
a
+
1
a+1
_
a
0
f(t)dt
(1) Express the value of
_
a
0
f(t)dt in terms of a.
(2) Find the maximum and minimum value of f(x) for 1 x 1.
272 Find lim
n
_

0
e
x
| sin nx|dx.
273 Find the area bounded by 0 y
x| sin x|
1+| cos x|
, 0 x n (n = 1, 2, ).
274 For real constant numbers a, b, c, d, consider the function f(x) = ax
3
+ bx
2
+ cx + d such
that f(1) = 0, f(1) = 0, f(x) 1 |x| for |x| 1.
Find f(x) for which
_
1
1
{f

(x) x}
2
dxis minimized.
275 The line y = mx has three intersection points with the curve y = |x(x 1)|. Find the value
of m such that the areas of two parts bounded by the line and the curve are equal.
276 Prove that
_

1
n
1
n
1
3

sin x
dx
3
2
(n = 1, 2, 3, ).
277 Find the fuction such that f(x) = x
2
x
_
2
_
1
0
|f(t)| dt
_1
2
.
278 Find the value of t such that


2
0
(sin x+t cos x)dx


2
0
(sin x+t cos x)
2
dx
is maximized.
279 If the function f(x) = 3x
2
+ 2ax +b satises
_
1
1
|f(x)| dx < 2, then show that f(x) = 0 has
distinct real roots.
http://www.artofproblemsolving.com/
This le was downloaded from the AoPS Math Olympiad Resources Page Page 2

Japan
Todays Calculation Of Integral
2008
280 Let p, q (p < q) be the x coordinates of curves x
2
+y
2
= 1, y 0 and y =
1
4x
.
(1) Find , such that cos

2
= p, cos

2
= q (0 < < , 0 < < ).
(2) Find the area bounded by the curves.
281 Find the maximum and minimum values of
_

0
(a sin x +b cos x)
3
dx for |a| 1, |b| 1.
282 g(x) is a dierentiable function for 0 x and g

(x) is a continuous function for 0 x .


Let f(x) = g(x) sin x. Find g(x) such that
_

0
{f(x)}
2
dx =
_

0
{f

(x)}
2
dx.
283 f(x) is a continuous function with the piriodicity of 2 and c is a positive constant number.
Find f(x) and c such that
_
2
0
f(t x) sin tdt = cf(x) with f(0) = 1 for all real numbers x.
284 f(x) is a continuous function which takes positive values for x 0. Find f(x) such that
_
x
0
f(t)dt = x
_
f(x) with f(1) =
1
2
.
285 Find the minimum value of
_
1
0
2
|2tx|+3
dtfor 0 x 4.
286 Evaluate
_
2008
2008
f

(x)+f

(x)
2008
x
+1
dx
287 For constant numbers a, b, let f(x) = e
x
(ax + b). Suppose that the curve y = f(x) passes
through two points (t, 1), (t +1, 1). Find the area of the part bounded by the curve y = f(x)
and the line y = 1.
288 Evaluate
_
1
0
(1 + 2008x
2008
)e
x
2008
dx.
289 Let p be a positive constant number, nd lim
a
_
p
0
|xsin(ax
2
)| dx.
290 Let the curve C : y = |x
2
+ 2x 3| and the line l passing through the point (3, 0) with
a slope m in x y plane. Suppose that C intersects to l at distinct two points other than
the point (3, 0), nd the value of m for which the area of the gure bounded by C and l is
minimized.
291 Consider the parabola C : y = x
2
in the x y plane. Let l
1
be the tangent line of C with
slope

3 and l
2
be the tangent line of C with negative slope which makes angle 45

with l
1
.
Find the area bounded by C, l
1
and l
2
.
292 Let x, y and t be real numbers such that
_
t
3
(2y 1)t
2
t + 2y 1 = 0
xt 2t + 2y = 0
In xy plane nd the area of the gure bounded by the part of curves above with x 0 and
the line y = 2.
http://www.artofproblemsolving.com/
This le was downloaded from the AoPS Math Olympiad Resources Page Page 3

Japan
Todays Calculation Of Integral
2008
293 Consider the parabolas C
a
: y =
1
4
x
2
ax+a
2
+a 2 and C : y =
1
4
x
2
+2 for real number
a in the x y plane.
(1) Find the equation of the locus of the vertex for C
a
.
(2) For a = 3, nd the slope of the two common tangent lines of C and C
a
, then the intersection
points of the lines.
(3) Suppose that C
a
intersects with C at two distinct points. Find the maximum area of the
gure bounded by C and C
a
.
294 Evaluate
_
ln 3
ln 2
ln(e
x
+1)
e
2x
dx.
295 Let f
n
(x) = x
n
(1 x)
n
, I
n
=
_
1
0
f
n
(x) dx (n = 1, 2, ).
(1) Find a polynomial g(x) such that f

n+1
(x) = (n + 1)f
n
(x)g(x).
(2) Find constant numbers A
n
, B
n
such that f

n+2
(x) = A
n
f
n+1
(x) +B
n
f
n
(x).
(3) Find A
n
I
n+1
+B
n
I
n
.
(4) Let J
n
= (2n + 1)!I
n
. Express J
n+1
in terms of J
n
.
(5) Find I
n
.
296 Let a
n
=
_

2
0
(1 sin t)
n
sin 2t dt.
(1) Find

n=1
a
n
.
(2) Find

n=1
an
n
.
(3) Find

n=1
(n + 1)(a
n
a
n+1
).
297 Evaluate
_

6
0
_
sin
3
2
x
_ _
cos
x
2
_
dx.
298 Let A be the region :
_
(x, y)|x 0, y 0,
_
x
a
_1
2
+
_
y
1a
_1
2
1
_
for 0 < a < 1.
(1) Find the area of A.
(2) Find the maximum volume of the solid obtained by revolving A around the x axis.
299 Let I
n
(x) =
_
x
1
(ln t)
n
dt (x > 0) for n = 1, 2, 3, .
(1) Prove by mathematical induction that I
n
(x) is expressed by I
n
(x) = xf
n
(ln x)+C
n
(n 1)
in terms of some polynomial f
n
(y) with degree n and some constant number C
n
.
(2) Express the constant term of f
n
(y) interms of n.
300 In Euclidean space, take the point N(0, 0, 1) on the sphere S with radius 1 centered in the
origin. For moving points P, Q on S such that NP = NQ and PNQ =
_
0 < <

2
_
,
consider the solid gure T in which the line segment PQ can be passed.
http://www.artofproblemsolving.com/
This le was downloaded from the AoPS Math Olympiad Resources Page Page 4

Japan
Todays Calculation Of Integral
2008
(1) Show that z coordinates of P, Q are equal.
(2) When P is on the palne z = h, express the length of PQ in terms of and h.
(3) Draw the outline of the cross section by cutting T by the plane z = h, then express the
area in terms of and h.
(4) Pay attention to the range for which h can be valued, express the volume V of T in terms
of , then nd the maximum V when let vary.
301 For the positive constant number a, let D be the part surrounded by the curve

x+

y =

a
and the line x +y = a.
(1) Draw the outline of D and nd the area.
(2) Find the volume of the solid by rotating D about the line x +y = a as the rotation axis.
302 Evaluate
_

0
|x1|
(x+1)(x
2
+1)
dx.
303 In the xy plane, nd the area of the region bounded by the parameterized curve as follows.
_
x = cos 2t
y = t sin t
(0 t 2)
304 Let , be real numbers with 0 and f(x) = x
2
( + )x + such that
_
1
1
f(x) dx = 1. Find the maximum value of
_

0
f(x) dx.
305 Find
_

0
(1

1 e
2x
) dx.
306 For positive real numbers a, b, two graphs of the function : x
a
and ln bx have a tangency of
point.
(1) Let (s, t) be the tangency of point, express this in terms of a, then express b as the
function of a.
(2) For h with 0 < h < s, denote the area as A(h) of the domain bounded by the line x = h
and two curves y = x
a
, y = ln bx.
Express lim
h0
A(h) in terms of a.
307 For real numbers a, b, c, let f(x) = ax
2
+ bx + c. Prove that :
_
1
1
(1 x
2
){f

(x)}
2
dx
6
_
1
1
{f(x)}
2
dx.
308 Let a be a positive constant number. For a positive integer n, dene a function I
n
(t) by
I
n
(t) =
_
t
0
x
n
e
ax
dx. Answer the following questions.
Note that you may use lim
t
t
n
e
at
= 0 without proof.
(1) Evaluate I
1
(t).
http://www.artofproblemsolving.com/
This le was downloaded from the AoPS Math Olympiad Resources Page Page 5

Japan
Todays Calculation Of Integral
2008
(2) Find the relation of I
n+1
(t), I
n
(t).
(3) Prove that there exists lim
t
I
n
(t) for all natural number n by using mathematical
induction.
(4) Find lim
t
I
n
(t).
309 (1) Calculate the indenite integral :
_
e
x
sin 2x dx.
(2) Evaluate the denite integral :
_

0
e
x
| sin 2x| dx.
310 Dene the function f(x) as : f(x) =
_
ln(1+x)
1+x
(x 0)
ln(1x)
1x
(1 x < 0)
1. Examine the variation of f(x) and nd the maximum and minimum value of f(x).
2. Find the value of a for which
_
e1
1
|f(x) a| dx is minimized for 0 a
1
e
.
311 Prove the following inequality.
_
1
0
2008
x
2008
dx
_
1
1
2008
2008
_
1
ln 2008
.
312 Let a, b be postive real numbers. For a real number t, denote by d(t) the distance between
the origin and the line (ae
t
)x + (be
t
)y = 1.
Let a, b vary with ab = 1, nd the minimum value of
_
1
0
1
d(t)
2
dt.
313 Find the area bounded by the graph of y =
3
_
x +

x
2
+ 1+
3
_
x

x
2
+ 1, the line xy1 =
0 and the x axis.
314 Evaluate
_

2
x
sin x
(1 +xcos x ln x + sin x) dx.
315 Evaluate
_
1
0
(x
2
3x+1)e
x
(x1)e
2x
x
(x+e
x
)
3
dx.
316 Evaluate
_
1
0
e
2x
e
2x
+4x
(e
x
+e
x
)
2
dx.
317 Evaluate
_
1
0
x+

x1
(

x+1)
2
dx.
318 Evaluate
_

3

4
1+cot x
e
x
sin x
dx.
319 Evaluate
_
1
1
xe
x
+e
x
+1
x
2
(e
x
+1)
2
dx.
320 Evaluate
_

0
e
xsin x
(x
2
cos x +xsin x + 1) dx.
http://www.artofproblemsolving.com/
This le was downloaded from the AoPS Math Olympiad Resources Page Page 6

Japan
Todays Calculation Of Integral
2008
321 Evaluate
_
1
1
x
2
e
x
x
2
e
2x
+2e
x
+2
(e
x
+1)
2
dx.
322 Evaluate
_
e
1
(x
4
4x
3
+9x
2
6x+8)e
x
x
3
dx.
323 Evaluate
_

2

4
_
_
sin x
x
+
_
x
sin x
cos x
_
dx.
324 Evaluate
_
e
1
_
1

xln x
+
_
ln x
x
_
dx.
325 Prove that
1
3
<
_
1
0
x
(sin x+cos x)
2
dx <
1
2
.
326 Evaluate
_
e
1
(x+1)(x+2ln x)
(x+ln x)
2
dx.
327 Let a, b be real numbers and C be the graph of the function y = e
a+bx
2
.
(1) Find the values of a, b such that C passes through the point P(1, 1) and the slope of the
tangent line of C at P is 2.
(2) For the values of a, b found in (1), nd the volume of the solid generated by the revolution
of the part which lies in the right side for the y axis in the gure bounded by the parabola
y = x
2
and the curve C about the y axis.
328 Let t be a negative real number and D be the part bounded by the curve y = 2
2x+2t
, the
curve y = 2
x+3t
and the y axis.
(1) Find the volume V (t) generated by rotation of D about the x axis.
(2) When t moves in the range of negative real number, nd the maximum value of V (t).
329 Let f(x), g(x) be continuous functions dened in the range of 0 x 1.
(1) Find f(x) such that f(x) =
_
1
0
e
x+t
f(t) dt.
(2) Find g(x) such that g(x) =
_
1
0
e
x+t
g(t) dt +x.
330 Find all real numbers x such that
_
x
0
t
2
sin(x t) dt = x
2
.
331 For a R, nd the minimum value of
_

2
0

sin 2x
1+sin
2
x
a cos x

dx.
332 Let f(x) be a function such that 1 f(x) = f(1 x) for 0 x 1.
Evaluate
_
1
0
f(x) dx.
333 Find the functions f(x), g(x) such that f(x) +
_
x
0
g(t) dt = sin x(cos x sin x), {f

(x)}
2
+
{g(x)}
2
= 1.
http://www.artofproblemsolving.com/
This le was downloaded from the AoPS Math Olympiad Resources Page Page 7

Japan
Todays Calculation Of Integral
2008
334 Evaluate
_
1
0
7x
3
+23x
2
+21x+15
(x
2
+1)(x+1)
2
dx.
335 For t > 0, prove that
1
2t
_
3t
t
xln x dx 2t ln 2t.
336 Let P
n
=
n
_
(3n)!
(2n)!
(n = 1, 2, ). Find the following limits.
(1) lim
n
Pn
n
.
(2) lim
n
_
n+2
n
_
Pn
.
337 Find lim
n
_

2
0

n
k=1
sin kxsin x dx.
338 Given a parameterized curve C : x = e
t
e
t
, y = e
3t
+e
3t
.
Find the area bounded by the curve C, the x axis and two lines x = 1.
339 Find the minimum area of the part bounded by the parabola y = a
3
x
2
a
4
x (a > 0) and the
line y = x.
340 Find the continuous function f(x) such that xf(x)
_
x
0
f(t) dt = x + ln(

x
2
+ 1 x) with
f(0) = ln 2.
341 Let c be a constant number. Find the real value of x such that
_
c
0
|t x| dt =
_
x
0
|t c| dt.
342 Prove the following inequality.
_
_
1
0
(x
2
+px +q) dx
_
2

_
1
0
(x
2
+px +q)
2
dx
1
180
343 Find all continuous positive functions f(x), for 0 x 1, such that
_
1
0
f(x) dx = 1
_
1
0
xf(x) dx =
_
1
0
x
2
f(x) dx =
2
where is given real number.
344 Find the value of k (0 < k < 5) such that
_

0
x
k
2+4x+3x
2
+5x
3
+3x
4
+4x
5
+2x
6
dx is minimal.
345 Given a continuous function f(x) such that
_
2
0
f(x) dx = 0.
Let S(x) = A
0
+A
1
cos x+B
1
sin x, nd constant numbers A
0
, A
1
and B
1
for which
_
2
0
{f(x)
S(x)}
2
dx is minimized.
http://www.artofproblemsolving.com/
This le was downloaded from the AoPS Math Olympiad Resources Page Page 8

Japan
Todays Calculation Of Integral
2008
346 Suppose that two curves C
1
: y = x
3
x, C
2
: y = (x a)
3
(x a) have two intersection
points.
Find the maximum area of the gure bounded by C
1
, C
2
.
347 For real positive numbers a, nd the minimum value of
_

0
_
x
a
+a sin x
_
2
dx.
348 Find lim
n
_
n+1
n
_ 1
n
2
_
n+2
n
_ 2
n
2
_
n+3
n
_ 3
n
2

_
n+n
n
_ n
n
2
(n = 1, 2, 3, ).
349 For real numbers a, b (ab = 0), evaluate
_

0
(a sin x +b cos x)
6
dx.
350 Evaluate
_
1
0
(1 x
2
)
5
2
dx.
351 Find the positive value of k for which
_

2
0
| cos x kx| dx is minimized.
352 Prove the following inequality.
1 lim
n
_
2
1
n
sin x
x( x)
dx < 3
.
353 Consider a parabola C : y =
1
4
x
2
and the point F(0, 1). For the origin O, take n points on
the parabola C, A
1
(x
1
, y
1
), A
2
(x
2
, y
2
), , A
n
(x
n
, y
n
) such that x
k
> 0 and OFA
k
=
k
2n
(k = 1, 2, 3, , n). Find lim
n
1
n

n
k=1
FA
k
.
354 Evaluate
_
2
2
x
4

4 x
2
dx.
355 For a positive number n, nd lim
n
n
2
_
1
e

1
n
x
n
ln x dx.
356 A continuous function f(x) satises that
d
dx
__
x
0
f(x +t) dt
_
= 0. Find the necessary and
sucient condition such that

n=1
f(2
n
) = 1.
357 For 0 < a < 1, let S(a) is the area of the gure bounded by three curves y = e
x
, y = e
1+a
1a
x
and y = e
2x
. Find lim
a0
S(a)
a
.
358 Let a be a postive constant number. Given a positive integer n, take an integer m such that
m
na

< m+ 1. Find lim


n
1
n
_
m
0
_
a
t
n
_
| sin t| dt.
359 Evaluate
_
1
1
2x
4
x
3
2x
2
+1
x
3
x+

1x
2
dx.
360 Evaluate
_
1
1
(1 +x)
1
2
(1 x)
3
2
dx.
http://www.artofproblemsolving.com/
This le was downloaded from the AoPS Math Olympiad Resources Page Page 9

Japan
Todays Calculation Of Integral
2008
361 Find the following indenite integrals.
(1)
_
sin x
3+sin
2
x
dx.
(2)
_
e
2x+e
x
dx.
362 Evaluate the following denite integrals.
(1)
_

2
0
xcos x dx.
(2)
_
2
1
1
x(x+1)
dx.
(3)
_
1
0
1
1+e
x
dx.
(4)
_

4
0
cos 2xcos 3x dx.
363 For t 0, let a
n
=
_
t
0
e
nx
dx (n = 0, 1, 2, 3).
(1) Show that a
3
3a
2
+ 3a
1
a
0
0.
(2) Show that e
t
a
0
+ (e
t
1)a
1
a
2
0.
364 Evaluate
_
1
0
x
2
(x 1)
2
e
2x
dx.
365 Let a, b be postive constant numbers.
Evaluate
_

2
0
a cos xb sin x
b sin x+a cos x
dx.
366 (1) Determine the constant numbers a, b, c, p, q, r, s such that the following equation is
equality.
4x = {a(x + 1)
2
+b(x + 1) +c}(x
2
+ 1)
2
+{(px +q)(x
2
+ 1) + (rx +s)}(x + 1)
3
.
(2) Evaluate the following denite integrals.
(a)
_
1
0
dx
x + 1
(b)
_
1
0
dx
(x + 1)
2
(c)
_
1
0
dx
(x + 1)
3
(d)
_
1
0
x
x
2
+ 1
dx (e)
_
1
0
dx
x
2
+ 1
(f)
_
1
0
x
(x
2
+ 1)
2
dx
(g)
_
1
0
dx
(x
2
+ 1)
2
(h)
_
1
0
4x
(x + 1)
3
(x
2
+ 1)
2
dx
367 Let f(x) = x
2
+
_
1
0
xf(t) dt +
_
2
1
f(t) dt.
Evaluate
_

2
0
f(x + sin x) dx.
368 For a real number a, evaluate
_
a
0
|x(x a
2
)| dx.
http://www.artofproblemsolving.com/
This le was downloaded from the AoPS Math Olympiad Resources Page Page 10

Japan
Todays Calculation Of Integral
2008
369 Calculate
e
e
e
e
_
e
e
e
1
xlnxln(lnx)ln(ln(lnx))
dx.
370 Calculate
_
/3
0
cos
2
x+1
cos x

cos x
dx.
371 Calculate
_
1
1
1
(1 +e
x
)(1 +x
2
)
dx.
372 Evaluate
_
1
0
e
x+e
x+e
x+e
x
dx.
373 Evaluate
_

2
0
e
x
_
cos (sin x) cos
2 x
2
+ sin (sin x) sin
2 x
2
_
dx.
374 Let n 2 be positive integers.
(1) Prove that nln n n + 1 <

n
k=1
ln k < (n + 1) ln n n + 1.
(2) Find lim
n
(n!)
1
nln n
.
375 Prove the following inequality.
1
n
<
_

2
0
1
(1+cos x)
n
dx <
n+5
n(n+1)
(n = 2, , ).
376 Evaluate
_

4
0
1+2 sin xsin
2
x
(1+x) cos
4
x
dx.
377 The line y = mx has 3 intersection points with the curve y = |x(x 1)|. Find the value of m
such that the area of the 2 regions bounded by the line and the curve are equal.
378 Evaluate
_

0
x| sin nx| dx (n = 1, 2, ).
379 Let , r be real numbers such that r > 1, r = 3, r = 4.
Find the values of , r such that lim
n

n
k=1
n

(n+k)
r
=
r3
(r1)(r4)
.
380 Find
_
1
1
n
C
k
(1 +x)
nk
(1 x)
k
dx (k = 0, 1, 2, n).
http://www.artofproblemsolving.com/
This le was downloaded from the AoPS Math Olympiad Resources Page Page 11

Japan
Todays Calculation Of Integral
2008
381 A function f(x) is dened as follows for x 0.
f(x) = sin
_
n
4
_
sin x (n x < (n + 1)) (n = 0, 1, 2, ).
Evaluate
_
100
0
f(x) dx.
382 (1) For a > 0, b 0, Compare
_
b+1
b
dx

x+a
,
1

a+b
,
1

a+b+1
.
(2) Find lim
n

n
k=1
1

n
2
+k
.
383 For a positive integer m, evaluate
_

2
0
cos
m
x sin 2mx dx.
384 Evaluate
_

3

4
(sin x + cos x)
_
e
x
sin x
dx.
385 Evaluate
_

2
4
0
(2 sin

x +

xcos

x) dx.
386 For a = 2007 2009, evaluate
_
1
0
6x+5a

x
4

x+a

x
dx.
387 Let l
1
, l
2
be the tangent and nomal line respectively at the point (p, ln(p +1)) on the curve
C : y = ln(x + 1). Denote by T
i
(i = 1, 2) the areas bounded by l
i
(i = 1, 2), C and the y
axis respectively. Find the limit lim
p0
T
2
T
1
.
388 For f(x) = sin
3
x, let p(x) be quadratic polynomial. Evaluate
_
2
0
p(x)f

(x) dx.
389 Find the values of t [0, 2] for which
_
2t
t3
2
x
2
dx is maximal and minimal.
390 Find the polynomials f(x), g(x) such that:
1

_
1
2
0
tf

(t)6g(t)

1t
2
dt = f(x) g(x) +x
6

_
1
2
0
8f(t)5g

(t)

1t
2
dt = 2f(x) 3g(x) x
2
+ 2x.
391 Evaluate
_

2
0
sin |2x a| dx for a real number a [0, ].
392 Evaluate
_
1
1
|x|
1+e
x
dx.
393 Let V (a) be the volume of the solid obtained by revolvong the region bounded by the curve
y = x

xsin ax (0 x

a
) and x axis around the x axis.
For a > 0, nd the minimum value of V (a) +V
_
1
a
_
.
http://www.artofproblemsolving.com/
This le was downloaded from the AoPS Math Olympiad Resources Page Page 12

Japan
Todays Calculation Of Integral
2008
394 Find lim
x0
1
x
_
x
x
(t sin 2006t + 2007t + 1004) dt.
395 3 points O(0, 0), P(a, a
2
), Q(b, b
2
) (a > 0, b > 0) are on the parabpla y = x
2
. Let S
1
be the area bounded by the line PQ and the parabola and let S
2
be the area of the triangle
OPQ.
Find the minimum value of
S
1
S
2
.
http://www.artofproblemsolving.com/
This le was downloaded from the AoPS Math Olympiad Resources Page Page 13

Japan
Todays Calculation Of Integral
2008
257 Evaluate
_

_
2008
0
x| sin x| dx
258 Find the volume of the solid formed by the revolution of the curve x = a(1+ cos ) cos , y =
a(1 + cos ) sin (a > 0, < ) about the x axis.
259 Evaluate
_

12
0
dx
(sin x+cos x)
4
260 Evaluate
_

3

4
(sin
3
cos
3
cos
2
)(sin +cos +cos
2
)
2007
sin
2009
cos
2009

d
261 Find the continuous function f(x) such that f(x) =
_
x
0
{f(t) cos t cos(t x)} dt.
262 Answer the following questions for positive integer n.
(1) Find the maximum value of f
n
(x) = x
n
e
x
for x 0.
(2) Show that lim
x
f
n
(x) = 0.
(3) Let I
n
(x) =
_
x
0
f
n
(t) dt, nd lim
x
I
n
(x).
263 Let F(t) =
1
t
_

2
t
0
| cos 2x| dx for 0 < t 1.
(1) Find lim
t0
F(t).
(2) Find the range of t such that F(t) 1.
264 Find the area of the gure surrounded by the locus of the point P inside the square with side
length a such that the distance to the center of the square is equal to the minimum distance
to the side of the square.
265 Supposed that f(x) has f

(x) and for any real numbers x, y,


_
x+y
y
f(t) dt =
_
x
0
{f(y) cos t +
f(t) cos y} dt holds.
(1) Express f(x + y) in terms of f(x), f(y). (2) Find f

_
x +

2
_
. Note that f
_

2
_
= 2. (3)
Find f(x).
266 Find the area of the region expressed by the system of inequality
x
2
4
+y
2
1,
_
1 +

3
2
_
x+y
1.
267 Let K be the curved surface obtained by rotating the parabola y =
3
4
x
2
about the y-
axis.Cut K by the plane H passing through the origin and forming angle 45

for the axis.


Find the volume of the solid surrounded by K and H.
Note that you are not allowed to use Double Integral for the problem.
268 Find the constant numbers u, v, s, t (s < t) such that
_
1
1
f(x) dx = uf(s) +vf(t) holds for
any polynomials f(x) = ax
3
+bx
2
+cx +d with the degree of 3 or less than.
http://www.artofproblemsolving.com/
This le was downloaded from the AoPS Math Olympiad Resources Page Page 1

Japan
Todays Calculation Of Integral
2008
269 For the curve C : y =
1
1+x
2
, Let A(, f()), B
_

, f
_

__
( > 0). Find the minimum
area bounded by the line segments OA, OB and C, where O is the origin.
Note that you are not allowed to use the integral formula of
1
1+x
2
for the problem.
270 Let f(x) be the continuous function at 0 x 1 such that
_
1
0
x
k
f(x) dx = 0 for integers
k = 0, 1, , n 1 (n 1).
(1) For all real numbers t, nd the minimum value of g(t) =
_
1
0
|x t|
n
dx.
(2) Show the following equation for all real real numbers t.
_
1
0
(x t)
n
f(x) dx =
_
1
0
x
n
f(x) dx
(3) Let M be the maximum value of the function |f(x)| for 0 x 1.
Show that

_
1
0
x
n
f(x) dx


M
2
n
(n+1)
271 For a positive constant number a, the function f(x) satises the following equation.
f(x) = xe

x
a
+
1
a+1
_
a
0
f(t)dt
(1) Express the value of
_
a
0
f(t)dt in terms of a.
(2) Find the maximum and minimum value of f(x) for 1 x 1.
272 Find lim
n
_

0
e
x
| sin nx|dx.
273 Find the area bounded by 0 y
x| sin x|
1+| cos x|
, 0 x n (n = 1, 2, ).
274 For real constant numbers a, b, c, d, consider the function f(x) = ax
3
+ bx
2
+ cx + d such
that f(1) = 0, f(1) = 0, f(x) 1 |x| for |x| 1.
Find f(x) for which
_
1
1
{f

(x) x}
2
dxis minimized.
275 The line y = mx has three intersection points with the curve y = |x(x 1)|. Find the value
of m such that the areas of two parts bounded by the line and the curve are equal.
276 Prove that
_

1
n
1
n
1
3

sin x
dx
3
2
(n = 1, 2, 3, ).
277 Find the fuction such that f(x) = x
2
x
_
2
_
1
0
|f(t)| dt
_1
2
.
278 Find the value of t such that


2
0
(sin x+t cos x)dx


2
0
(sin x+t cos x)
2
dx
is maximized.
279 If the function f(x) = 3x
2
+ 2ax +b satises
_
1
1
|f(x)| dx < 2, then show that f(x) = 0 has
distinct real roots.
http://www.artofproblemsolving.com/
This le was downloaded from the AoPS Math Olympiad Resources Page Page 2

Japan
Todays Calculation Of Integral
2008
280 Let p, q (p < q) be the x coordinates of curves x
2
+y
2
= 1, y 0 and y =
1
4x
.
(1) Find , such that cos

2
= p, cos

2
= q (0 < < , 0 < < ).
(2) Find the area bounded by the curves.
281 Find the maximum and minimum values of
_

0
(a sin x +b cos x)
3
dx for |a| 1, |b| 1.
282 g(x) is a dierentiable function for 0 x and g

(x) is a continuous function for 0 x .


Let f(x) = g(x) sin x. Find g(x) such that
_

0
{f(x)}
2
dx =
_

0
{f

(x)}
2
dx.
283 f(x) is a continuous function with the piriodicity of 2 and c is a positive constant number.
Find f(x) and c such that
_
2
0
f(t x) sin tdt = cf(x) with f(0) = 1 for all real numbers x.
284 f(x) is a continuous function which takes positive values for x 0. Find f(x) such that
_
x
0
f(t)dt = x
_
f(x) with f(1) =
1
2
.
285 Find the minimum value of
_
1
0
2
|2tx|+3
dtfor 0 x 4.
286 Evaluate
_
2008
2008
f

(x)+f

(x)
2008
x
+1
dx
287 For constant numbers a, b, let f(x) = e
x
(ax + b). Suppose that the curve y = f(x) passes
through two points (t, 1), (t +1, 1). Find the area of the part bounded by the curve y = f(x)
and the line y = 1.
288 Evaluate
_
1
0
(1 + 2008x
2008
)e
x
2008
dx.
289 Let p be a positive constant number, nd lim
a
_
p
0
|xsin(ax
2
)| dx.
290 Let the curve C : y = |x
2
+ 2x 3| and the line l passing through the point (3, 0) with
a slope m in x y plane. Suppose that C intersects to l at distinct two points other than
the point (3, 0), nd the value of m for which the area of the gure bounded by C and l is
minimized.
291 Consider the parabola C : y = x
2
in the x y plane. Let l
1
be the tangent line of C with
slope

3 and l
2
be the tangent line of C with negative slope which makes angle 45

with l
1
.
Find the area bounded by C, l
1
and l
2
.
292 Let x, y and t be real numbers such that
_
t
3
(2y 1)t
2
t + 2y 1 = 0
xt 2t + 2y = 0
In xy plane nd the area of the gure bounded by the part of curves above with x 0 and
the line y = 2.
http://www.artofproblemsolving.com/
This le was downloaded from the AoPS Math Olympiad Resources Page Page 3

Japan
Todays Calculation Of Integral
2008
293 Consider the parabolas C
a
: y =
1
4
x
2
ax+a
2
+a 2 and C : y =
1
4
x
2
+2 for real number
a in the x y plane.
(1) Find the equation of the locus of the vertex for C
a
.
(2) For a = 3, nd the slope of the two common tangent lines of C and C
a
, then the intersection
points of the lines.
(3) Suppose that C
a
intersects with C at two distinct points. Find the maximum area of the
gure bounded by C and C
a
.
294 Evaluate
_
ln 3
ln 2
ln(e
x
+1)
e
2x
dx.
295 Let f
n
(x) = x
n
(1 x)
n
, I
n
=
_
1
0
f
n
(x) dx (n = 1, 2, ).
(1) Find a polynomial g(x) such that f

n+1
(x) = (n + 1)f
n
(x)g(x).
(2) Find constant numbers A
n
, B
n
such that f

n+2
(x) = A
n
f
n+1
(x) +B
n
f
n
(x).
(3) Find A
n
I
n+1
+B
n
I
n
.
(4) Let J
n
= (2n + 1)!I
n
. Express J
n+1
in terms of J
n
.
(5) Find I
n
.
296 Let a
n
=
_

2
0
(1 sin t)
n
sin 2t dt.
(1) Find

n=1
a
n
.
(2) Find

n=1
an
n
.
(3) Find

n=1
(n + 1)(a
n
a
n+1
).
297 Evaluate
_

6
0
_
sin
3
2
x
_ _
cos
x
2
_
dx.
298 Let A be the region :
_
(x, y)|x 0, y 0,
_
x
a
_1
2
+
_
y
1a
_1
2
1
_
for 0 < a < 1.
(1) Find the area of A.
(2) Find the maximum volume of the solid obtained by revolving A around the x axis.
299 Let I
n
(x) =
_
x
1
(ln t)
n
dt (x > 0) for n = 1, 2, 3, .
(1) Prove by mathematical induction that I
n
(x) is expressed by I
n
(x) = xf
n
(ln x)+C
n
(n 1)
in terms of some polynomial f
n
(y) with degree n and some constant number C
n
.
(2) Express the constant term of f
n
(y) interms of n.
300 In Euclidean space, take the point N(0, 0, 1) on the sphere S with radius 1 centered in the
origin. For moving points P, Q on S such that NP = NQ and PNQ =
_
0 < <

2
_
,
consider the solid gure T in which the line segment PQ can be passed.
http://www.artofproblemsolving.com/
This le was downloaded from the AoPS Math Olympiad Resources Page Page 4

Japan
Todays Calculation Of Integral
2008
(1) Show that z coordinates of P, Q are equal.
(2) When P is on the palne z = h, express the length of PQ in terms of and h.
(3) Draw the outline of the cross section by cutting T by the plane z = h, then express the
area in terms of and h.
(4) Pay attention to the range for which h can be valued, express the volume V of T in terms
of , then nd the maximum V when let vary.
301 For the positive constant number a, let D be the part surrounded by the curve

x+

y =

a
and the line x +y = a.
(1) Draw the outline of D and nd the area.
(2) Find the volume of the solid by rotating D about the line x +y = a as the rotation axis.
302 Evaluate
_

0
|x1|
(x+1)(x
2
+1)
dx.
303 In the xy plane, nd the area of the region bounded by the parameterized curve as follows.
_
x = cos 2t
y = t sin t
(0 t 2)
304 Let , be real numbers with 0 and f(x) = x
2
( + )x + such that
_
1
1
f(x) dx = 1. Find the maximum value of
_

0
f(x) dx.
305 Find
_

0
(1

1 e
2x
) dx.
306 For positive real numbers a, b, two graphs of the function : x
a
and ln bx have a tangency of
point.
(1) Let (s, t) be the tangency of point, express this in terms of a, then express b as the
function of a.
(2) For h with 0 < h < s, denote the area as A(h) of the domain bounded by the line x = h
and two curves y = x
a
, y = ln bx.
Express lim
h0
A(h) in terms of a.
307 For real numbers a, b, c, let f(x) = ax
2
+ bx + c. Prove that :
_
1
1
(1 x
2
){f

(x)}
2
dx
6
_
1
1
{f(x)}
2
dx.
308 Let a be a positive constant number. For a positive integer n, dene a function I
n
(t) by
I
n
(t) =
_
t
0
x
n
e
ax
dx. Answer the following questions.
Note that you may use lim
t
t
n
e
at
= 0 without proof.
(1) Evaluate I
1
(t).
http://www.artofproblemsolving.com/
This le was downloaded from the AoPS Math Olympiad Resources Page Page 5

Japan
Todays Calculation Of Integral
2008
(2) Find the relation of I
n+1
(t), I
n
(t).
(3) Prove that there exists lim
t
I
n
(t) for all natural number n by using mathematical
induction.
(4) Find lim
t
I
n
(t).
309 (1) Calculate the indenite integral :
_
e
x
sin 2x dx.
(2) Evaluate the denite integral :
_

0
e
x
| sin 2x| dx.
310 Dene the function f(x) as : f(x) =
_
ln(1+x)
1+x
(x 0)
ln(1x)
1x
(1 x < 0)
1. Examine the variation of f(x) and nd the maximum and minimum value of f(x).
2. Find the value of a for which
_
e1
1
|f(x) a| dx is minimized for 0 a
1
e
.
311 Prove the following inequality.
_
1
0
2008
x
2008
dx
_
1
1
2008
2008
_
1
ln 2008
.
312 Let a, b be postive real numbers. For a real number t, denote by d(t) the distance between
the origin and the line (ae
t
)x + (be
t
)y = 1.
Let a, b vary with ab = 1, nd the minimum value of
_
1
0
1
d(t)
2
dt.
313 Find the area bounded by the graph of y =
3
_
x +

x
2
+ 1+
3
_
x

x
2
+ 1, the line xy1 =
0 and the x axis.
314 Evaluate
_

2
x
sin x
(1 +xcos x ln x + sin x) dx.
315 Evaluate
_
1
0
(x
2
3x+1)e
x
(x1)e
2x
x
(x+e
x
)
3
dx.
316 Evaluate
_
1
0
e
2x
e
2x
+4x
(e
x
+e
x
)
2
dx.
317 Evaluate
_
1
0
x+

x1
(

x+1)
2
dx.
318 Evaluate
_

3

4
1+cot x
e
x
sin x
dx.
319 Evaluate
_
1
1
xe
x
+e
x
+1
x
2
(e
x
+1)
2
dx.
320 Evaluate
_

0
e
xsin x
(x
2
cos x +xsin x + 1) dx.
http://www.artofproblemsolving.com/
This le was downloaded from the AoPS Math Olympiad Resources Page Page 6

Japan
Todays Calculation Of Integral
2008
321 Evaluate
_
1
1
x
2
e
x
x
2
e
2x
+2e
x
+2
(e
x
+1)
2
dx.
322 Evaluate
_
e
1
(x
4
4x
3
+9x
2
6x+8)e
x
x
3
dx.
323 Evaluate
_

2

4
_
_
sin x
x
+
_
x
sin x
cos x
_
dx.
324 Evaluate
_
e
1
_
1

xln x
+
_
ln x
x
_
dx.
325 Prove that
1
3
<
_
1
0
x
(sin x+cos x)
2
dx <
1
2
.
326 Evaluate
_
e
1
(x+1)(x+2ln x)
(x+ln x)
2
dx.
327 Let a, b be real numbers and C be the graph of the function y = e
a+bx
2
.
(1) Find the values of a, b such that C passes through the point P(1, 1) and the slope of the
tangent line of C at P is 2.
(2) For the values of a, b found in (1), nd the volume of the solid generated by the revolution
of the part which lies in the right side for the y axis in the gure bounded by the parabola
y = x
2
and the curve C about the y axis.
328 Let t be a negative real number and D be the part bounded by the curve y = 2
2x+2t
, the
curve y = 2
x+3t
and the y axis.
(1) Find the volume V (t) generated by rotation of D about the x axis.
(2) When t moves in the range of negative real number, nd the maximum value of V (t).
329 Let f(x), g(x) be continuous functions dened in the range of 0 x 1.
(1) Find f(x) such that f(x) =
_
1
0
e
x+t
f(t) dt.
(2) Find g(x) such that g(x) =
_
1
0
e
x+t
g(t) dt +x.
330 Find all real numbers x such that
_
x
0
t
2
sin(x t) dt = x
2
.
331 For a R, nd the minimum value of
_

2
0

sin 2x
1+sin
2
x
a cos x

dx.
332 Let f(x) be a function such that 1 f(x) = f(1 x) for 0 x 1.
Evaluate
_
1
0
f(x) dx.
333 Find the functions f(x), g(x) such that f(x) +
_
x
0
g(t) dt = sin x(cos x sin x), {f

(x)}
2
+
{g(x)}
2
= 1.
http://www.artofproblemsolving.com/
This le was downloaded from the AoPS Math Olympiad Resources Page Page 7

Japan
Todays Calculation Of Integral
2008
334 Evaluate
_
1
0
7x
3
+23x
2
+21x+15
(x
2
+1)(x+1)
2
dx.
335 For t > 0, prove that
1
2t
_
3t
t
xln x dx 2t ln 2t.
336 Let P
n
=
n
_
(3n)!
(2n)!
(n = 1, 2, ). Find the following limits.
(1) lim
n
Pn
n
.
(2) lim
n
_
n+2
n
_
Pn
.
337 Find lim
n
_

2
0

n
k=1
sin kxsin x dx.
338 Given a parameterized curve C : x = e
t
e
t
, y = e
3t
+e
3t
.
Find the area bounded by the curve C, the x axis and two lines x = 1.
339 Find the minimum area of the part bounded by the parabola y = a
3
x
2
a
4
x (a > 0) and the
line y = x.
340 Find the continuous function f(x) such that xf(x)
_
x
0
f(t) dt = x + ln(

x
2
+ 1 x) with
f(0) = ln 2.
341 Let c be a constant number. Find the real value of x such that
_
c
0
|t x| dt =
_
x
0
|t c| dt.
342 Prove the following inequality.
_
_
1
0
(x
2
+px +q) dx
_
2

_
1
0
(x
2
+px +q)
2
dx
1
180
343 Find all continuous positive functions f(x), for 0 x 1, such that
_
1
0
f(x) dx = 1
_
1
0
xf(x) dx =
_
1
0
x
2
f(x) dx =
2
where is given real number.
344 Find the value of k (0 < k < 5) such that
_

0
x
k
2+4x+3x
2
+5x
3
+3x
4
+4x
5
+2x
6
dx is minimal.
345 Given a continuous function f(x) such that
_
2
0
f(x) dx = 0.
Let S(x) = A
0
+A
1
cos x+B
1
sin x, nd constant numbers A
0
, A
1
and B
1
for which
_
2
0
{f(x)
S(x)}
2
dx is minimized.
http://www.artofproblemsolving.com/
This le was downloaded from the AoPS Math Olympiad Resources Page Page 8

Japan
Todays Calculation Of Integral
2008
346 Suppose that two curves C
1
: y = x
3
x, C
2
: y = (x a)
3
(x a) have two intersection
points.
Find the maximum area of the gure bounded by C
1
, C
2
.
347 For real positive numbers a, nd the minimum value of
_

0
_
x
a
+a sin x
_
2
dx.
348 Find lim
n
_
n+1
n
_ 1
n
2
_
n+2
n
_ 2
n
2
_
n+3
n
_ 3
n
2

_
n+n
n
_ n
n
2
(n = 1, 2, 3, ).
349 For real numbers a, b (ab = 0), evaluate
_

0
(a sin x +b cos x)
6
dx.
350 Evaluate
_
1
0
(1 x
2
)
5
2
dx.
351 Find the positive value of k for which
_

2
0
| cos x kx| dx is minimized.
352 Prove the following inequality.
1 lim
n
_
2
1
n
sin x
x( x)
dx < 3
.
353 Consider a parabola C : y =
1
4
x
2
and the point F(0, 1). For the origin O, take n points on
the parabola C, A
1
(x
1
, y
1
), A
2
(x
2
, y
2
), , A
n
(x
n
, y
n
) such that x
k
> 0 and OFA
k
=
k
2n
(k = 1, 2, 3, , n). Find lim
n
1
n

n
k=1
FA
k
.
354 Evaluate
_
2
2
x
4

4 x
2
dx.
355 For a positive number n, nd lim
n
n
2
_
1
e

1
n
x
n
ln x dx.
356 A continuous function f(x) satises that
d
dx
__
x
0
f(x +t) dt
_
= 0. Find the necessary and
sucient condition such that

n=1
f(2
n
) = 1.
357 For 0 < a < 1, let S(a) is the area of the gure bounded by three curves y = e
x
, y = e
1+a
1a
x
and y = e
2x
. Find lim
a0
S(a)
a
.
358 Let a be a postive constant number. Given a positive integer n, take an integer m such that
m
na

< m+ 1. Find lim


n
1
n
_
m
0
_
a
t
n
_
| sin t| dt.
359 Evaluate
_
1
1
2x
4
x
3
2x
2
+1
x
3
x+

1x
2
dx.
360 Evaluate
_
1
1
(1 +x)
1
2
(1 x)
3
2
dx.
http://www.artofproblemsolving.com/
This le was downloaded from the AoPS Math Olympiad Resources Page Page 9

Japan
Todays Calculation Of Integral
2008
361 Find the following indenite integrals.
(1)
_
sin x
3+sin
2
x
dx.
(2)
_
e
2x+e
x
dx.
362 Evaluate the following denite integrals.
(1)
_

2
0
xcos x dx.
(2)
_
2
1
1
x(x+1)
dx.
(3)
_
1
0
1
1+e
x
dx.
(4)
_

4
0
cos 2xcos 3x dx.
363 For t 0, let a
n
=
_
t
0
e
nx
dx (n = 0, 1, 2, 3).
(1) Show that a
3
3a
2
+ 3a
1
a
0
0.
(2) Show that e
t
a
0
+ (e
t
1)a
1
a
2
0.
364 Evaluate
_
1
0
x
2
(x 1)
2
e
2x
dx.
365 Let a, b be postive constant numbers.
Evaluate
_

2
0
a cos xb sin x
b sin x+a cos x
dx.
366 (1) Determine the constant numbers a, b, c, p, q, r, s such that the following equation is
equality.
4x = {a(x + 1)
2
+b(x + 1) +c}(x
2
+ 1)
2
+{(px +q)(x
2
+ 1) + (rx +s)}(x + 1)
3
.
(2) Evaluate the following denite integrals.
(a)
_
1
0
dx
x + 1
(b)
_
1
0
dx
(x + 1)
2
(c)
_
1
0
dx
(x + 1)
3
(d)
_
1
0
x
x
2
+ 1
dx (e)
_
1
0
dx
x
2
+ 1
(f)
_
1
0
x
(x
2
+ 1)
2
dx
(g)
_
1
0
dx
(x
2
+ 1)
2
(h)
_
1
0
4x
(x + 1)
3
(x
2
+ 1)
2
dx
367 Let f(x) = x
2
+
_
1
0
xf(t) dt +
_
2
1
f(t) dt.
Evaluate
_

2
0
f(x + sin x) dx.
368 For a real number a, evaluate
_
a
0
|x(x a
2
)| dx.
http://www.artofproblemsolving.com/
This le was downloaded from the AoPS Math Olympiad Resources Page Page 10

Japan
Todays Calculation Of Integral
2008
369 Calculate
e
e
e
e
_
e
e
e
1
xlnxln(lnx)ln(ln(lnx))
dx.
370 Calculate
_
/3
0
cos
2
x+1
cos x

cos x
dx.
371 Calculate
_
1
1
1
(1 +e
x
)(1 +x
2
)
dx.
372 Evaluate
_
1
0
e
x+e
x+e
x+e
x
dx.
373 Evaluate
_

2
0
e
x
_
cos (sin x) cos
2 x
2
+ sin (sin x) sin
2 x
2
_
dx.
374 Let n 2 be positive integers.
(1) Prove that nln n n + 1 <

n
k=1
ln k < (n + 1) ln n n + 1.
(2) Find lim
n
(n!)
1
nln n
.
375 Prove the following inequality.
1
n
<
_

2
0
1
(1+cos x)
n
dx <
n+5
n(n+1)
(n = 2, , ).
376 Evaluate
_

4
0
1+2 sin xsin
2
x
(1+x) cos
4
x
dx.
377 The line y = mx has 3 intersection points with the curve y = |x(x 1)|. Find the value of m
such that the area of the 2 regions bounded by the line and the curve are equal.
378 Evaluate
_

0
x| sin nx| dx (n = 1, 2, ).
379 Let , r be real numbers such that r > 1, r = 3, r = 4.
Find the values of , r such that lim
n

n
k=1
n

(n+k)
r
=
r3
(r1)(r4)
.
380 Find
_
1
1
n
C
k
(1 +x)
nk
(1 x)
k
dx (k = 0, 1, 2, n).
http://www.artofproblemsolving.com/
This le was downloaded from the AoPS Math Olympiad Resources Page Page 11

Japan
Todays Calculation Of Integral
2008
381 A function f(x) is dened as follows for x 0.
f(x) = sin
_
n
4
_
sin x (n x < (n + 1)) (n = 0, 1, 2, ).
Evaluate
_
100
0
f(x) dx.
382 (1) For a > 0, b 0, Compare
_
b+1
b
dx

x+a
,
1

a+b
,
1

a+b+1
.
(2) Find lim
n

n
k=1
1

n
2
+k
.
383 For a positive integer m, evaluate
_

2
0
cos
m
x sin 2mx dx.
384 Evaluate
_

3

4
(sin x + cos x)
_
e
x
sin x
dx.
385 Evaluate
_

2
4
0
(2 sin

x +

xcos

x) dx.
386 For a = 2007 2009, evaluate
_
1
0
6x+5a

x
4

x+a

x
dx.
387 Let l
1
, l
2
be the tangent and nomal line respectively at the point (p, ln(p +1)) on the curve
C : y = ln(x + 1). Denote by T
i
(i = 1, 2) the areas bounded by l
i
(i = 1, 2), C and the y
axis respectively. Find the limit lim
p0
T
2
T
1
.
388 For f(x) = sin
3
x, let p(x) be quadratic polynomial. Evaluate
_
2
0
p(x)f

(x) dx.
389 Find the values of t [0, 2] for which
_
2t
t3
2
x
2
dx is maximal and minimal.
390 Find the polynomials f(x), g(x) such that:
1

_
1
2
0
tf

(t)6g(t)

1t
2
dt = f(x) g(x) +x
6

_
1
2
0
8f(t)5g

(t)

1t
2
dt = 2f(x) 3g(x) x
2
+ 2x.
391 Evaluate
_

2
0
sin |2x a| dx for a real number a [0, ].
392 Evaluate
_
1
1
|x|
1+e
x
dx.
393 Let V (a) be the volume of the solid obtained by revolvong the region bounded by the curve
y = x

xsin ax (0 x

a
) and x axis around the x axis.
For a > 0, nd the minimum value of V (a) +V
_
1
a
_
.
http://www.artofproblemsolving.com/
This le was downloaded from the AoPS Math Olympiad Resources Page Page 12

Japan
Todays Calculation Of Integral
2008
394 Find lim
x0
1
x
_
x
x
(t sin 2006t + 2007t + 1004) dt.
395 3 points O(0, 0), P(a, a
2
), Q(b, b
2
) (a > 0, b > 0) are on the parabpla y = x
2
. Let S
1
be the area bounded by the line PQ and the parabola and let S
2
be the area of the triangle
OPQ.
Find the minimum value of
S
1
S
2
.
http://www.artofproblemsolving.com/
This le was downloaded from the AoPS Math Olympiad Resources Page Page 13

Japan
Todays Calculation Of Integral
2010
522 Find lim
a
1
a
2
_
a
0
ln(1 +e
x
)dx.
523 Prove the following inequality.
ln

2009 +

2010

2008 +

2009
<
_

2009

2008
_
1 e
x
2
x
dx <

2009

2008
524 Evaluate the following denite integral.
2
2009
_
1
0
x
1004
(1 x)
1004
dx
_
1
0
x
1004
(1 x
2010
)
1004
dx
525 Let a, b be real numbers satisfying
_
1
0
(ax+b)
2
dx = 1. Determine the values of a, b for which
_
1
0
3x(ax +b) dx is maximized.
526 For a function satisfying f

(x) > 0 for a x b, let F(x) =


_
b
a
|f(t) f(x)| dt. For what
value of x is F(x) is minimized?
527 Let n, m be positive integers and , be real numbers. Prove the following equations.
(1)
_

(x )(x ) dx =
1
6
( )
3
(2)
_

(x )
n
(x ) dx =
n!
(n+2)!
( )
n+2
(3)
_

(x )
n
(x )
m
dx = (1)
m n!m!
(n+m+1)!
( )
n+m+1
528 Consider a function f(x) = xe
x
3
dened on any real numbers.
(1) Examine the variation and convexity of f(x) to draw the garph of f(x).
(2) For a positive number C, let D
1
be the region bounded by y = f(x), the x-axis and x = C.
Denote V
1
(C) the volume obtained by rotation of D
1
about the x-axis. Find lim
C
V
1
(C).
(3) Let M be the maximum value of y = f(x) for x 0. Denote D
2
the region bounded by
y = f(x), the y-axis and y = M. Find the volume V
2
obtained by rotation of D
2
about the
y-axis.
529 Prove that the following inequality holds for each natural number n.
_
2
0
n

k=1
_
sin kx
k
_
2
dx <
61
144

http://www.artofproblemsolving.com/
This le was downloaded from the AoPS Math Olympiad Resources Page Page 1

Japan
Todays Calculation Of Integral
2010
530 Answer the following questions.
(1) By setting x +

x
2
1 = t, nd the indenite integral
_
x
2
1 dx.
(2) Given two points P(p, q) (p > 1, q > 0) and A(1, 0) on the curve x
2
y
2
= 1. Find the
area S of the gure bounded by two lines OA, OP and the curve in terms of p.
(3) Let S =

2
. Express p, q in terms of .
531 (1) Let f(x) be a continuous function dened on [a, b], it is known that there exists some c
such that
_
b
a
f(x) dx = (b a)f(c) (a < c < b)
Explain the fact by using graph. Note that you dont need to prove the statement.
(2) Let f(x) = a
0
+a
1
x +a
2
x
2
+ +a
n
x
n
,
Prove that there exists such that
f(sin ) = a
0
+
a
1
2
+
a
3
3
+ +
a
n
n + 1
, 0 < <

2
.
532 For a curve C : y = x

9 x
2
(x 0),
(1) Find the maximum value of the function.
(2) Find the area of the gure bounded by the curve C and the x-axis.
(3) Find the volume of the solid by revolution of the gure in (2) around the y-axis.
Please nd the volume without using cylindrical shells for my students.
Last Edited.
533 Let C be the circle with radius 1 centered on the origin. Fix the endpoint of the string with
length 2 on the point A(1, 0) and put the other end point P on the point P
0
(1, 2). From
this situation, when we twist the string around C by moving the point P in anti clockwise
with the string streched tightly, nd the length of the curve that the point P draws from
sarting point P
0
to reaching point A.
534 Find the indenite integral
_
x
3
(x1)
3
(x2)
dx.
535 Let C be the parameterized curve for a given positive number r and 0 t ,
C :
_
x = 2r(t sin t cos t)
y = 2r sin
2
t
When the point P moves on the curve C,
http://www.artofproblemsolving.com/
This le was downloaded from the AoPS Math Olympiad Resources Page Page 2

Japan
Todays Calculation Of Integral
2010
(1) Find the magnitude of acceleralation of the point P at time t.
(2) Find the length of the locus by which the point P sweeps for 0 t .
(3) Find the volume of the solid by rotation of the region bounded by the curve C and the
x-axis about the x-axis.
Edited.
536 Evaluate
_

4
0
x+sin x
1+cos x
dx.
537 Evaluate
_

6
0

1+sin x
cos x
dx.
538 Evaluate
_

2
1
x
2
+1
x

x
4
+1
dx.
539 Evaluate
_

4
0
sin
2
x
cos
3
x
dx.
540 Evaluate
_
e
1
3

x
x(

x+
3

x)
dx.
541 Find the functions f(x), g(x) satisfying the following equations.
(1) f

(x) = 2f(x) + 10, f(0) = 0


(2)
_
x
0
u
3
g(u)du = x
4
+g(x)
542 Find continuous functions f(x), g(x) which takes positive value for any real number x, sat-
isfying g(x) =
_
x
0
f(t) dt and {f(x)}
2
{g(x)}
2
= 1.
543 Let y be the function of x satisfying the dierential equation y

y = 2 sin x.
(1) Let y = e
x
u sin x, nd the dierential equation with which the function u with respect
to x satises.
(2) If y(0) = 3, y

(0) = 0, then determine y.


544 (1) Evaluate
_

3
(x
2
1)dx,
_

3
(x 1)
2
dx,
_

3
(x + 1)
2
dx.
(2) If a linear function f(x) saties
_

3
(x 1)f(x)dx = 5

3,
_

3
(x + 1)f(x)dx = 3

3,
then we have f(x) = A (x 1) + B (x + 1), thus we have f(x) = C .
545 (1) Evaluate
_
1
0
xe
x
2
dx.
(2) Let I
n
=
_
1
0
x
2n1
e
x
2
dx. Express I
n+1
in terms of I
n
.
546 Find the minimum value of
_

0
_
x a
b

cos x
_
2
dx.
547 Find the minimum value of
_
1
0
|e
x
a|dx ( < a < ).
http://www.artofproblemsolving.com/
This le was downloaded from the AoPS Math Olympiad Resources Page Page 3

Japan
Todays Calculation Of Integral
2010
548 For f(x) = e
x
2
cos
x
2
, evaluate

n=0
_

f(x)f(x 2n)dx (n = 0, 1, 2, ).
549 Let f(x) be a function dened on [0, 1]. For n = 1, 2, 3, , a polynomial P
n
(x) is dened
by P
n
(x) =

n
k=0
n
C
k
f
_
k
n
_
x
k
(1 x)
nk
. Prove that lim
n
_
1
0
P
n
(x)dx =
_
1
0
f(x)dx.
550 Evaluate
_

2
0
dx
(1+cos x)
2
.
551 In the coordinate plane, nd the area of the region bounded by the curve C : y =
x+1
x
2
+1
and
the line L : y = 1.
552 Find the positive value of a such that the curve C
1
: x =
_
2y
2
+
25
2
tangent to the parabola
C
2
: y = ax
2
, then nd the equation of the tangent line of C
1
at the point of tangency.
553 Find the continuous function such that f(x) =
e
2x
2(e1)
_
1
0
e
y
f(y)dy+
_
1
2
0
f(y)dy+
_
1
2
0
sin
2
(y)dy.
554 Use
d
dx
ln(2x +

4x
2
+ 1),
d
dx
(x

4x
2
+ 1) to evaluate
_
1
0

4x
2
+ 1dx.
555 For
1
e
< t < 1, nd the minimum value of
_
1
0
|xe
x
tx|dx.
556 Prove the following inequality.
3

_
4
0
x
cos
2
xcos
2
(tan x) cos
2
(tan(tan x)) cos
2
(tan(tan(tan x)))
dx <
4

Last Edited.
Sorry, I have changed the problem.
kunny
557 Find the folllowing limit.
lim
n
(2n + 1)
_
1
0
x
n1
sin
_

2
x
_
dx
(n + 1)
2
_
1
0
x
n1
cos
_

2
x
_
dx
(n = 1, 2, ).
558 For a positive constant t, let , be the roots of the quadratic equation x
2
+t
2
x 2t = 0.
Find the minimum value of
_
2
1
_
_
x +
1

2
_
_
x +
1

2
_
+
1

_
dx.
559 In xyz space, consider two points P(1, 0, 1), Q(1, 1, 0). Let S be the surface generated
by rotation the line segment PQ about x axis. Answer the following questions.
(1) Find the volume of the solid bounded by the surface S and two planes x = 1 and x = 1.
http://www.artofproblemsolving.com/
This le was downloaded from the AoPS Math Olympiad Resources Page Page 4

Japan
Todays Calculation Of Integral
2010
(2) Find the cross-section of the solid in (1) by the plane y = 0 to sketch the gure on the
palne y = 0.
(3) Evaluate the denite integral
_
1
0

t
2
+ 1 dt by substitution t =
e
s
e
s
2
.
Then use this to nd the area of (2).
560 Let K be the gure bounded by the graph of function y =
x

1x
2
, x axis and the line x =
1
2
.
(1) Find the volume V
1
of the solid generated by rotation of K around x axis.
(2) Find the volume V
2
of the solid generated by rotation of K around y axis.
Please solve question (2) without using the shell method for Japanese High School Students
those who dont learn it.
561 Evaluate
_
1
1
1 + 2x
2
+ 3x
4
+ 4x
6
+ 5x
8
+ 6x
10
+ 7x
12
_
(1 +x
2
)(1 +x
4
)(1 +x
6
)
dx.
562 (1) Show the following inequality for every natural number k.
1
2(k + 1)
<
_
1
0
1 x
k +x
dx <
1
2k
(2) Show the following inequality for every natural number m, n such that m > n.
mn
2(m+ 1)(n + 1)
< log
m
n

m

k=n+1
1
k
<
mn
2mn
563 Determine the pair of constant numbers a, b, c such that for a quadratic function f(x) =
x
2
+ax +b, the following equation is identity with respect to x.
f(x + 1) = c
_
1
0
(3x
2
+ 4xt)f

(t)dt
.
564 In the coordinate plane with O(0, 0), consider the function C : y =
1
2
x +
_
1
4
x
2
+ 2 and two
distinct points P
1
(x
1
, y
1
), P
2
(x
2
, y
2
) on C.
(1) Let H
i
(i = 1, 2) be the intersection points of the line passing through P
i
(i = 1, 2),
parallel to x axis and the line y = x. Show that the area of OP
1
H
1
and OP
2
H
2
are equal.
(2) Let x
1
< x
2
. Express the area of the gure bounded by the part of x
1
x x
2
for C
and line segments P
1
O, P
2
O in terms of y
1
, y
2
.
http://www.artofproblemsolving.com/
This le was downloaded from the AoPS Math Olympiad Resources Page Page 5

Japan
Todays Calculation Of Integral
2010
565 Prove that f(x) =
_
1
0
e
|tx|
t(1 t)dt has maximal value at x =
1
2
.
566 In the coordinate space, consider the cubic with vertices O(0, 0, 0), A(1, 0, 0), B(1, 1, 0), C(0, 1, 0), D(0, 0, 1), E(1, 0, 1), F(1, 1, 1), G(0, 1, 1).
Find the volume of the solid generated by revolution of the cubic around the diagonal OF as
the axis of rotation.
567 Let a be a positive real numbers. In the coordinate plane denote by S the area of the gure
bounded by the curve y = sin x (0 x ) and the x-axis and denote T by the area of the
gure bounded by the curves y = sin x
_
0 x

2
_
, y = a cos x
_
0 x

2
_
and the x-axis.
Find the value of a such that S : T = 3 : 1.
568 Throw n balls in to 2n boxes.Suppose each ball comes into each box with equal probability
of entering in any boxes. Let p
n
be the probability such that any box has ball less than or
equal to one. Find the limit lim
n
ln pn
n
569 In the coordinate plane, denote by S(a) the area of the region bounded by the line passing
through the point (1, 2) with the slope a and the parabola y = x
2
. When a varies in the
range of 0 a 6, nd the value of a such that S(a) is minimized.
570 Let f(x) = 1 cos x xsin x.
(1) Show that f(x) = 0 has a unique solution in 0 < x < .
(2) Let J =
_

0
|f(x)|dx. Denote by the solution in (1), express J in terms of sin .
(3) Compare the size of J dened in (2) with

2.
571 Evaluate
_

0
xsin
3
x
sin
2
x+8
dx.
572 For integer n, a
n
is dined by a
n
=
_

4
0
(cos x)
n
dx.
(1) Find a
2
, a
1
.
(2) Find the relation of a
n
and a
n2
.
(3) Prove that a
2n
= b
n
+c
n
for some rational number b
n
, c
n
, then nd c
n
for n < 0.
573 Find the area of the gure bounded by three curves
C
1
: y = sin x
_
0 x <

2
_
C
2
: y = cos x
_
0 x <

2
_
C
3
: y = tan x
_
0 x <

2
_
.
574 Let n be a positive integer. Prove that x
n
e
1x
n! for x 0,
http://www.artofproblemsolving.com/
This le was downloaded from the AoPS Math Olympiad Resources Page Page 6

Japan
Todays Calculation Of Integral
2010
575 For a function f(x) =
_
4
x
x
log
4
(1 + tan t)dt
_
0 x

8
_
, answer the following questions.
(1) Find f

(x).
(2) Find the n th term of the sequence a
n
such that a
1
= f(0), a
n+1
= f(a
n
) (n =
1, 2, 3, ).
576 For a function f(x) = (ln x)
2
+2 ln x, let C be the curve y = f(x). Denote A(a, f(a)), B(b, f(b)) (a <
b) the points of tangency of two tangents drawn from the origin O to C and the curve C.
Answer the following questions.
(1) Examine the increase and decrease, extremal value and inection point , then draw the
approximate garph of the curve C.
(2) Find the values of a, b.
(3) Find the volume by a rotation of the gure bounded by the part from the point A to the
point B and line segments OA, OB around the y-axis.
577 Prove the following inequality for any integer N 4.
N

p=4
p
2
+ 2
(p 2)
4
< 5
578 Find the range of k for which the following inequality holds for 0 x 1.
_
x
0
dt
_
(3 +t
2
)
3
k
_
x
0
dt

3 +t
2
If necessary, you may use ln 3 = 1.10.
579 Let a be a positive real number. Find lim
n
(n+1)
a
+(n+2)
a
++(n+n)
a
1
a
+2
a
++n
a
580 Let k be a positive constant number. Denote , (0 < < ) the x coordinates of the curve
C : y = kx
2
(x 0) and two lines l : y = kx +
1
k
, m : y = kx +
1
k
.Find the minimum area
of the part bounded by the curve C and two lines l, m.
581 For real numer c for which cx
2
ln(1 + x
2
) for all real numbers x, nd the value of c such
that the area of the gure bounded by two curves y = cx
2
and y = ln(1 + x
2
) and two lines
x = 1, x = 1 is 4.
582 Prove the following inequality.

4
_
3
2
+

2 <
_
2
0
_
1
1
2
sin
2
x dx <

3
4

http://www.artofproblemsolving.com/
This le was downloaded from the AoPS Math Olympiad Resources Page Page 7

Japan
Todays Calculation Of Integral
2010
583 Find the values of k such that the areas of the three parts bounded by the graph of y =
x
4
+ 2x
2
and the line y = k are all equal.
584 Find lim
x
_
_
x
0

1 +e
2t
dt e
x
_
.
585 Evaluate
_
ln 2
0
(x ln 2)e
2 ln(1+e
x
)+x+ln 2
dx.
586 Evaluate
_
1
0
x
14
x
2
+1
dx.
587 Evaluate
_
1
0
(x
2
+3x)e
x
(x
2
3x)e
x
+2

1+x(e
x
+e
x
)
dx.
588 Evaluate
_

2
0
e
xe
x
{(x + 1)e
x
(cos x + sin x) + cos x sin x}dx.
589 Evaluate
_
1
0
x
{(2x1)

x
2
+x+1+(2x+1)

x
2
x+1}

x
4
+x
2
+1
dx.
590 Evaluate
_

8
0
(cos +sin )
3
2 (cos sin )
3
2

cos 2
d.
591 Let a, b, c be real numbers such that a b c 1.
Prove the following inequality:
_
1
0
{(1 ax)
3
+ (1 bx)
3
+ (1 cx)
3
3x} dx ab +bc +ca
3
2
(a +b +c)
3
4
abc.
592 Prove the following inequality.

2
4

1
2

1
4
ln 2 <
_
4
0
ln cos x dx <
3
8
+
1
2
ln (3 + 2

2)
593 For a positive integer m, prove the following ineqaulity.
0
_
1
0
_
x + 1
_
x
2
+ 2xcos
2
2m+1
+ 1
_
dx 1.
1996 Osaka University entrance exam
594 In the x-y plane, two variable points P, Q stay in P(2t, 2t
2
+2t), Q(t +2, 3t +2) at the
time t. Let denote t
0
as the time such that PQ = 0. When t varies in the range of 0 t t
0
,
nd the area of the region swept by the line segment PQ in the x-y plane.
595 Evaluate
_

6

4 sin x

3 cos xsin x

dx.
2009 Kumamoto University entrance exam/Medicine
http://www.artofproblemsolving.com/
This le was downloaded from the AoPS Math Olympiad Resources Page Page 8

Japan
Todays Calculation Of Integral
2010
596 Find the minimum value of
_

2
0
|a sin 2x cos
2
x|dx (a > 0).
2009 Shimane University entrance exam/Medicine
597 In space given a board shaped the equilateral triangle PQR with vertices P
_
1,
1
2
, 0
_
, Q
_
1,
1
2
, 0
_
, R
_
1
4
, 0,

3
4
_
.
When S is revolved about the z-axis, nd the volume of the solid generated by the whole
points through which S passes.
1984 Tokyo University entrance exam/Science
598 For a constant a, denote C(a) the part x 1 of the curve y =

x
2
1 +
a
x
.
(1) Find the maximum value a
0
of a such that C(a) is contained to lower part of y = x, or
y < x.
(2) For 0 < <

2
, nd the volume V () of the solid V obtained by revoloving the gure
bounded by C(a
0
) and three lines y = x, x = 1, x =
1
cos
about the x-axis.
(3) Find lim

2
0
V ().
1992 Tokyo University entrance exam/Science, 2nd exam
599 Evaluate
_

6
0
e
x
(sin x+cos x+cos 3x)
cos
2
2x
dx.
created by kunny
600 Evaluate
_
a
a
_
x +
1
sin x+
1
e
x
e
x
_
dx (a > 0).
created by kunny
601 Evaluate
_

4
0
(tan x)
3
2
dx.
created by kunny
602 Prove the following inequality.
e 1
n + 1

_
e
1
(log x)
n
dx
(n + 1)e + 1
(n + 1)(n + 2)
(n = 1, 2, )
1994 Kyoto University entrance exam/Science
603 Find the minimum value of
_
1
0
{

x (a +bx)}
2
dx.
Please solve the problem without using partial dierentiation for those who dont learn it.
1961 Waseda University entrance exam/Science and Technology
http://www.artofproblemsolving.com/
This le was downloaded from the AoPS Math Olympiad Resources Page Page 9

Japan
Todays Calculation Of Integral
2010
604 Let r be a positive integer. Determine the value of a for which the limit value lim
n

n
k=1
k
r
n
a
has a non zero nite value, then nd the limit value.
1956 Tokyo Institute of Technology entrance exam
605 Let f(x) be a dierentiable function. Find the following limit value:
lim
n
_
n
k
_
_
f
_
x
n
_
f(0)
_
k
.
Especially, for f(x) = (x )(x ) nd the limit value above.
1956 Tokyo Institute of Technology entrance exam
606 Find the area of the part bounded by two curves y =

x,

x +

y = 1 and the x-axis.


1956 Tokyo Institute of Technology entrance exam
607 On the coordinate plane, Let C be the graph of y = (ln x)
2
(x > 0) and for > 0, denote
L() be the tangent line of C at the point (, (ln )
2
).
(1) Draw the graph.
(2) Let n() be the number of the intersection points of C and L(). Find n().
(3) For 0 < < 1, let S() be the area of the region bounded by C, L() and the x-axis.
Find S().
2010 Tokyo Institute of Technology entrance exam, Second Exam.
608 For a > 0, nd the minimum value of
_
1
0
ax
2
+(a
2
+2a)x+2a
2
2a+4
(x+a)(x+2)
dx.
2010 Gakusyuin University entrance exam/Science
609 Prove that for positive number t, the function F(t) =
_
t
0
sin x
1+x
2
dx always takes positive number.
1972 Tokyo University of Education entrance exam
610 Evaluate
_
a
2
x
a
1xa
x
ln a
(x
a
1)
2
dx.
proposed by kunny
611 Let g(t) be the minimum value of f(x) = x2
x
in t x t + 1. Evaluate
_
2
0
g(t)dt.
2010 Kumamoto University entrance exam/Science
612 For f(x) =
1
x
(x > 0), prove the following inequality.
f
_
t +
1
2
_

_
t+1
t
f(x) dx
1
6
_
f(t) + 4f
_
t +
1
2
_
+f(t + 1)
_
http://www.artofproblemsolving.com/
This le was downloaded from the AoPS Math Olympiad Resources Page Page 10

Japan
Todays Calculation Of Integral
2010
613 Find the area of the part, in the x-y plane, enclosed by the curve |ye
2x
6e
x
8| = (e
x

2)(e
x
4).
2010 Tokyo University of Agriculture and Technology entrance exam
614 Evaluate
_
1
0
{x(1 x)}
3
2
dx.
2010 Hirosaki University School of Medicine entrance exam
615 For 0 a 2, nd the minimum value of
_
2
0

1
1+e
x

1
1+e
a

dx.
2010 Kyoto Institute of Technology entrance exam/Textile e.t.c.
616 Evaluate
_
3
1
ln(x+1)
x
2
dx.
2010 Hirosaki University entrance exam
617 Let y = f(x) be a function of the graph of broken line connected by points (1, 0), (0, 1), (1, 4)
in the x -y plane. Find the minimum value of
_
1
1
{f(x) (a|x| +b)}
2
dx.
2010 Tohoku University entrance exam/Economics, 2nd exam
618 Find the minimu value of
1

_

2

2
{xcos t + (1 x) sin t}
2
dt.
2010 Ibaraki University entrance exam/Science
619 Consider a function f(x) =
sin x
9+16 sin
2
x
_
0 x

2
_
. Let a be the value of x for which f(x) is
maximized.
Evaluate
_
2
a
f(x) dx.
2010 Saitama University entrance exam/Mathematics
Last Edited
620 Let a, b be real numbers. Suppose that a function f(x) satises f(x) = a sin x + b cos x +
_

f(t) cos t dt and has the maximum value 2 for x . Find the minimum value of
_

{f(x)}
2
dx.
2010 Chiba University entrance exam
621 Find the limit lim
n
1
n

n
k=1
k ln
_
n
2
+(k1)
2
n
2
+k
2
_
.
2010 Yokohama National University entrance exam/Engineering, 2nd exam
622 For 0 < k < 2, consider two curves C
1
: y = sin 2x (0 x ), C
2
: y = k cos x (0 x ).
Denote by S(k) the sum of the areas of four parts enclosed by C
1
, C
2
and two lines x =
0, x = . Find the minimum value of S(k).
2010 Nagoya Institute of Technology entrance exam
http://www.artofproblemsolving.com/
This le was downloaded from the AoPS Math Olympiad Resources Page Page 11

Japan
Todays Calculation Of Integral
2010
623 Find the continuous function satisfying the following equation.
_
x
0
f(t)dt +
_
x
0
tf(x t)dt = e
x
1.
1978 Shibaura Institute of Technology entrance exam
624 Find the continuous function f(x) such that the following equation holds for any real number
x.
_
x
0
sin t f(x t)dt = f(x) sin x.
1977 Keio University entrance exam/Medicine
625 Find lim
t0
1
t
3
_
t
2
0
e
x
sin
x
t
dx (t = 0).
2010 Kumamoto University entrance exam/Medicine
626 Find lim
a+0
_
1
a
xln x
(1+x)
3
dx.
2010 Nara Medical University entrance exam
627 Evaluate
_

3

4
(2 sin +1) cos
3

(sin
2
+1)
2
d.
Proposed by kunny
628 (1) Evaluate the following denite integrals.
(a)
_

2
0
cos
2
xsin x dx
(b)
_

2
0
( 2x) cos x dx
(c)
_

2
0
xcos
3
x dx
(2) Let a be a positive constant. Find the area of the cross section cut by the plane z =
sin
_
0

2
_
of the solid such that
x
2
+y
2
+z
2
a
2
, x
2
+y
2
ax, z 0
, then nd the volume of the solid.
1984 Yamanashi Medical University entrance exam
Please slove the problem without multi integral or arcsine function for Japanese high school
students aged 17-18 those who dont study them.
Thanks in advance.
kunny
http://www.artofproblemsolving.com/
This le was downloaded from the AoPS Math Olympiad Resources Page Page 12

Japan
Todays Calculation Of Integral
2010
629 Evaluate
_

0
1
e
x
(1+e
4x
)
dx.
630 Evaluate
_

0
ln(1+e
4x
)
e
x
dx.
631 Evaluate
_

2
(x
2
+

x
4
1)
_
1

x
2
+1
+
1

x
2
1
_
dx.
Proposed by kunny
632 Find lim
n
_
1
0
| sin nx|
3
dx (n = 1, 2, ).
2010 Kyoto Institute of Technology entrance exam/Textile, 2nd exam
633 Let f(x) be a dierentiable function. Find the value of x for which
{f(x)}
2
+ (e + 1)f(x) + 1 +e
2
2
_
x
0
f(t)dt 2f(x)
_
x
0
f(t)dt + 2
__
x
0
f(t)dt
_
2
is minimized.
1978 Tokyo Medical College entrance exam
634 Prove that :
_

e
1
(ln x)
n
dx = (1)
n1
n! +

e
n

m=0
(1)
nm
n!
m!
_
1
2
_
m
(n = 1, 2, )
2010 Miyazaki University entrance exam/Medicine
635 Suppose that a function f(x) dened in 1 < x < 1 satises the following properties (i) ,
(ii), (iii).
(i) f

(x) is continuous.
(ii) When 1 < x < 0, f

(x) < 0, f

(0) = 0, when 0 < x < 1, f

(x) > 0.
(iii) f(0) = 1
Let F(x) =
_
x
0
_
1 +{f

(t)}
2
dt (1 < x < 1). If F(sin ) = c (c : constant) holds for

2
< <

2
, then nd f(x).
1975 Waseda University entrance exam/Science and Technology
636 Let a > 1 be a constant. In the xy-plane, let A(a, 0), B(a, ln a) and C be the intersection
point of the curve y = ln x and the x-axis. Denote by S
1
the area of the part bounded by the
x-axis, the segment BA and the curve y = ln x
(1) For 1 b a, let D(b, ln b). Find the value of b such that the area of quadrilateral
ABDC is the closest to S
1
and nd the area S
2
.
http://www.artofproblemsolving.com/
This le was downloaded from the AoPS Math Olympiad Resources Page Page 13

Japan
Todays Calculation Of Integral
2010
(2) Find lim
a
S
2
S
1
.
1992 Tokyo University entrance exam/Science
637 For a non negative integer n, set t I
n
=
_

4
0
tan
n
x dx to answer the following questions:
(1) Calculate I
n+2
+I
n
.
(2) Evaluate the values of I
1
, I
2
and I
3
.
1978 Niigata university entrance exam
638 Let (a, b) be a point on the curve y =
x
1+x
(x 0). Denote U the volume of the gure enclosed
by the curve , the x axis and the line x = a, revolved around the the x axis and denote V
the volume of the gure enclosed by the curve , the y axis and th line y = b, revolved around
the y axis. Whats the relation of U and V ?
1978 Chuo university entrance exam/Science and Technology
639 Evaluate
_
1
0
(x + 3)

xe
x
dx.
640 Evaluate
_

4
0
1
1sin x
_
cos x
1+cos x+sin x
dx.
Own
641 Evaluate
_
e
e
e
e
e
_
ln(ln(ln x)) +
1
(ln x) ln(ln x)
_
dx.
Own
642 Evaluate
_
6
0
(tan
2
2x)

cos 2x + 2
(cos
2
x)

cos 2x
dx.
Own
643 Evaluate
_

0
x
_
1 + sin
3
x
{(3 cos x + 4 sin x) sin
2
x + 4}dx.
Own
http://www.artofproblemsolving.com/
This le was downloaded from the AoPS Math Olympiad Resources Page Page 14

Japan
Todays Calculation Of Integral
2010
644 For a constant p such that
_
p
1
e
x
dx = 1, prove that
__
p
1
e
x
cos x dx
_
2
+
__
p
1
e
x
sin x dx
_
2
>
1
2
.
Own
645 Prove the following inequality.
_
1
1
e
x
+e
x
e
e
e
x
dx < e
1
e
Own
646 Evaluate
_

0
a
x
cos bx dx,
_

0
a
x
sin bx dx (a > 0, a = 1, b N
+
)
Own
647 Evaluate
_

0
xp
x
cos qx dx,
_

0
xp
x
sin qx dx (p > 0, p = 1, q N
+
)
Own
648 Consider a function real-valued function with C

-class on R such that:


(a) f(0) =
df
dx
(0) = 0,
d
2
f
dx
2
(0) = 0.
(b) For x = 0, f(x) > 0.
Judge whether the following integrals (i), (ii) converge or diverge, justify your answer.
(i)
_ _
|x
1
|
2
+|x
2
|
2
1
dx
1
dx
2
f(x
1
) +f(x
2
)
.
(ii)
_ _
|x
1
|
2
+|x
2
|
2
+|x
3
|
2
1
dx
1
dx
2
dx
3
f(x
1
) +f(x
2
) +f(x
3
)
.
2010 Kyoto University, Master Course in Mathematics
http://www.artofproblemsolving.com/
This le was downloaded from the AoPS Math Olympiad Resources Page Page 15

Japan
Todays Calculation Of Integral
2010
649 Let f
n
(x, y) =
n
r cos r+n
2
r
3
(r =
_
x
2
+y
2
),
I
n
=
_ _
r1
f
n
(x, y) dxdy (n 2).
Find lim
n
I
n
.
2009 Tokyo Institute of Technology, Master Course in Mathematics
650 Find the values of p, q, r (1 < p < q < r < 1) such that for any polynomials with degree 2,
the following equation holds:
_
p
1
f(x) dx
_
q
p
f(x) dx +
_
r
q
f(x) dx
_
1
r
f(x) dx = 0.
1995 Hitotsubashi University entrance exam/Law, Economics etc.
651 Find
lim
n
_
2n
0
e
2x

x 2
x + 1
2

dx.
1985 Tohoku University entrance exam/Mathematics, Physics, Chemistry, Biology
652 Let a, b, c be positive real numbers such that b
2
> ac. Evaluate
_

0
dx
ax
4
+ 2bx
2
+c
.
1981 Tokyo University, Master Course
653 Sign me in. If is not to late ! s:) img src=SMILIES
P
ATH/smile.gifalt = :)title =
Smile/ ><! s :) > Afunctionf(x)definedinx 0 satises lim
x
f(x)
x
= 1.
Find
_

0
{f(x) f

(x)}e
x
dx.
1997 Hokkaido University entrance exam/Science
654 655 Find the area of the region of the points such that the total of three tangent lines can be
drawn to two parabolas y = x x
2
, y = a(x x
2
) (a 2) in such a way that there existed
the points of tangency in the rst quadrant.
656 Find lim
n
n
_

2
0
1
(1+cos x)
n
dx (n = 1, 2, ).
657 A sequence a
n
is dened by
_
a
n+1
an
(1 +| sin x|)dx = (n + 1)
2
(n = 1, 2, ), a
1
= 0.
Find lim
n
an
n
3
.
http://www.artofproblemsolving.com/
This le was downloaded from the AoPS Math Olympiad Resources Page Page 16

Japan
Todays Calculation Of Integral
2010
658 Consider a parameterized curve C : x = e
t
cos t, y = e
t
sin t
_
0 t

2
_
.
(1) Find the length L of C.
(2) Find the area S of the region enclosed by the x, y axis and C.
Please solve the problem without using the formula of area for polar coordinate for Japanese
High School Students who dont study it in High School.
1997 Kyoto University entrance exam/Science
659 Evaluate
_
1
0
ln(x+2)
x+1
dx.
660 Let a, b be given positive constants.
Evaluate
_
1
0
ln (x +a)
x+a
(x +b)
x+b
(x +a)(x +b)
dx.
Own
661 Consider a sequence 1
0.01
, 2
0.02
, 2
0.02
, 3
0.03
, 3
0.03
, 3
0.03
, 4
0.04
, 4
0.04
, 4
0.04
, 4
0.04
, .
(1) Find the 36th term.
(2) Find
_
x
2
ln x dx.
(3) Let A be the product of from the rst term to the 36th term. How many digits does A
have integer part?
If necessary, you may use the fact 2.0 < ln 8 < 2.1, 2.1 < ln 9 < 2.2, 2.30 < ln 10 < 2.31.
2010 National Defense Medical College Entrance Exam, Problem 4
662 In xyz space, let A be the solid generated by a rotation of the gure, enclosed by the curve
y = 2 2x
2
and the x-axis about the y-axis.
(1) When the solid is cut by the plane x = a (|a| 1), nd the inequality which expresses
the gure of the cross-section.
(2) Denote by L the distance between the point (a, 0, 0) and the point on the perimeter of
the cross-section found in (1), nd the maximum value of L.
(3) Find the volume of the solid by a rotation of the solid A about the x-axis.
1987 Sophia University entrance exam/Science and Technology
663 Given are the curve y = x
2
+ x 2 and a curve which is obtained by tranfering the curve
symmetric with respect to the point (p, 2p). Let p change in such a way that these two curves
intersects, nd the maximum area of the part bounded by these curves.
1978 Nagasaki University entrance exam/Economics
http://www.artofproblemsolving.com/
This le was downloaded from the AoPS Math Olympiad Resources Page Page 17

Japan
Todays Calculation Of Integral
2010
664 For a positive integer n, let I
n
=
_

2
|x|
_
cos nx dx.
Find I
1
+I
2
+I
3
+I
4
.
1992 University of Fukui entrance exam/Medicine
665 Find lim
n
_

0
x| sin 2nx|dx (n = 1, 2, ).
1992 Japan Womens University entrance exam/Physics, Mathematics
666 Let f(x) be a function dened in 0 < x <

2
satisfying:
(i) f
_

6
_
= 0
(ii) f

(x) tan x =
_
x

6
2 cos t
sin t
dt.
Find f(x).
1987 Sapporo Medical University entrance exam
667 Let a > 1, 0 x

4
. Find the volume of the solid generated by a rotation of the part
bounded by two curves y =

2 sin x

sin 2x+a
, y =
1

sin 2x+a
about the x-axis.
1993 Hiroshima Un iversity entrance exam/Science
668 Consider two curves y = sin x, y = sin 2x in 0 x 2.
(1) Let (, ) (0 < < ) be the intersection point of the curves. If sin x sin 2x has
a local minimum at x = x
1
and a local maximum at x = x
2
, then nd the values of
cos x
1
, cos x
1
cos x
2
.
(2) Find the area enclosed by the curves, then nd the volume of the part generated by a
rotation of the part of x for the gure about the line y = 1.
2011 KyorinUniversity entrance exam/Medicine
669 Find the dierentiable function dened in x > 0 such that
_
f(x)
1
f
1
(t)dt =
1
3
(x
3
2
8).
http://www.artofproblemsolving.com/
This le was downloaded from the AoPS Math Olympiad Resources Page Page 18

Japan
Todays Calculation Of Integral
2011
673 Let f(x) =
_
x
0
1
1+t
2
dt. For 1 x < 1, nd cos
_
2f
__
1+x
1x
__
.
2011 Ritsumeikan University entrance exam/Science and Technology
674 Evaluate
_
1
0
x
2
+5
(x+1)
2
(x2)
dx.
2011 Doshisya University entrance exam/Science and Technology
675 In the coordinate plane with the origin O, consider points P(t+2, 0), Q(0, 2t
2
2t+4) (t
0). If the y-coordinate of Q is nonnegative, then nd the area of the region swept out by the
line segment PQ.
2011 Ritsumeikan University entrance exam/Pharmacy
676 Let f(x) = cos
4
x + 3 sin
4
x. Evaluate
_

2
0
|f

(x)|dx.
2011 Tokyo University of Science entrance exam/Management
677 Let a, b be positive real numbers with a < b. Dene the denite integrals I
1
, I
2
, I
3
by
I
1
=
_
b
a
sin (x
2
) dx, I
2
=
_
b
a
cos (x
2
)
x
2
dx, I
3
=
_
b
a
sin (x
2
)
x
4
dx.
(1) Find the value of I
1
+
1
2
I
2
in terms of a, b.
(2) Find the value of I
2

3
2
I
3
in terms of a, b.
(3) For a positive integer n, dene K
n
=
_

2(n+1)

2n
sin (x
2
) dx +
3
4
_

2(n+1)

2n
sin (x
2
)
x
4
dx.
Find the value of lim
n
2n

2nK
n
.
2011 Tokyo University of Science entrance exam/Information Sciences, Applied Chemistry,
Mechanical Enginerring, Civil Enginerring
678 Evaluate
_

0
_
1 +
n

k=1
k cos kx
_
2
dx (n = 1, 2, ).
2011 Doshisya University entrance exam/Life Medical Sciences
679 Find

3n
k=1
1

1
0
x(1x)
k
dx
.
2011 Hosei University entrance exam/Design and Enginerring
680 Let a > 0. Evaluate
_
a
0
x
2
_
1
x
a
_
a
dx.
2011 Keio University entrance exam/Science and Technology
681 Evaluate
_

2
0

1 2 sin 2x + 3 cos
2
x dx.
2011 University of Occupational and Environmental Health/Medicineentrance exam
http://www.artofproblemsolving.com/
This le was downloaded from the AoPS Math Olympiad Resources Page Page 1

Japan
Todays Calculation Of Integral
2011
682 On the x-y plane, 3 half-lines y = 0, (x 0), y = xtan (x 0), y =

3x (x 0)
intersect with the circle with the center the origin O, radius r 1 at A, B, C respectively.
Note that

6


3
.
If the area of quadrilateral OABC is one third of the area of the regular hexagon which
inscribed in a circle with radius 1, then evaluate
_

3

6
r
2
d.
2011 Waseda University of Education entrance exam/Science
683 Evaluate
_
1
2
0
(x + 1)

1 2x
2
dx.
2011 Kyoto University entrance exam/Science, Problem 1B
684 On the xy plane, nd the area of the gure bounded by the graphs of y = x and y =

3
4
x
2
3

2.
2011 Kyoto University entrance exam/Science, Problem 3
685 Suppose that a cubic function with respect to x, f(x) = ax
3
+ bx
2
+ cx + d satises all of 3
conditions:
f(1) = 1, f(1) = 1,
_
1
1
(bx
2
+cx +d) dx = 1
.
Find f(x) for which I =
_
1
2
1
{f

(x)}
2
dx is minimized, the nd the minimum value.
2011 Tokyo University entrance exam/Humanities, Problem 1
686 Let L be a positive constant. For a point P(t, 0) on the positive part of the x axis on the
coordinate plane, denote Q(u(t), v(t)) the point at which the point reach starting from P
proceeds bydistance L in counter-clockwise on the perimeter of a circle passing the point P
with center O.
(1) Find u(t), v(t).
(2) For real number a with 0 < a < 1, nd f(a) =
_
1
a
_
{u

(t)}
2
+ {v

(t)}
2
dt.
(3) Find lim
a+0
f(a)
ln a
.
2011 Tokyo University entrance exam/Science, Problem 3
687 (1) Let x > 0, y be real numbers. For variable t, nd the dierence of Maximum and minimum
value of the quadratic function f(t) = xt
2
+yt in 0 t 1.
(2) Let S be the domain of the points (x, y) in the coordinate plane forming the following
condition:
For x > 0 and all real numbers t with 0 t 1 , there exists real number z for which
0 xt
2
+yt +z 1 .
http://www.artofproblemsolving.com/
This le was downloaded from the AoPS Math Olympiad Resources Page Page 2

Japan
Todays Calculation Of Integral
2011
Sketch the outline of S.
(3) Let V be the domain of the points (x, y, z) in the coordinate space forming the following
condition:
For 0 x 1 and for all real numbers t with 0 t 1, 0 xt
2
+yt +z 1 holds.
Find the volume of V .
2011 Tokyo University entrance exam/Science, Problem 6
688 For a real number x, let f(x) =
_

2
0
| cos t xsin 2t| dt.
(1) Find the minimum value of f(x).
(2) Evaluate
_
1
0
f(x) dx.
2011 Tokyo Institute of Technology entrance exam, Problem 2
689 Let C : y = x
2
+ax +b be a parabola passing through the point (1, 1). Find the minimum
volume of the gure enclosed by C and the x axis by a rotation about the x axis.
Proposed by kunny
690 Find the maximum value of f(x) =
_
1
0
t sin(x +t) dt.
691 Let a be a constant. In the xy palne, the curve C
1
: y =
ln x
x
touches C
2
: y = ax
2
. Find the
volume of the solid generated by a rotation of the part enclosed by C
1
, C
2
and the x axis
about the x axis.
2011 Yokohama National Universty entrance exam/Engineering
692 Evaluate
_

12
0
tan
2
x3
3 tan
2
x1
dx.
created by kunny
693 Evaluate
_

0
4
_
1 + | cos x| dx.
created by kunny
694 Prove the following inequality:
_
e
1
(ln x)
2009
x
2
dx >
1
2010 2011 2012
created by kunny
http://www.artofproblemsolving.com/
This le was downloaded from the AoPS Math Olympiad Resources Page Page 3

Japan
Todays Calculation Of Integral
2011
695 For a positive integer n, let
S
n
=
_
1
0
1 (x)
n
1 +x
dx, T
n
=
n

k=1
(1)
k1
k(k + 1)
Answer the following questions:
(1) Show the following inequality.

S
n

_
1
0
1
1 +x
dx

1
n + 1
(2) Express T
n
2S
n
in terms of n.
(3) Find the limit lim
n
T
n
.
696 Let P(x), Q(x) be polynomials such that :
_
2
0
{P(x)}
2
dx = 14,
_
2
0
P(x)dx = 4,
_
2
0
{Q(x)}
2
dx = 26,
_
2
0
Q(x)dx = 2.
Find the maximum and the minimum value of
_
2
0
P(x)Q(x)dx.
697 Find the volume of the solid of the domain expressed by the inequality x
2
x y x,
generated by a rotation about the line y = x.
698 For a positive integer n, let denote C
n
the gure formed by the inside and perimeter of the
circle with center the origin, radius n on the x-y plane.
Denote by N(n) the number of a unit square such that all of unit square, whose x, y coordi-
nates of 4 vertices are integers, and the vertices are included in C
n
.
Prove that lim
n
N(n)
n
2
= .
699 Find the volume of the part bounded by z = x +y, z = x
2
+y
2
in the xyz space.
700 Evaluate
_

0
x
2
cos
2
x xsin x cos x 1
(1 +xsin x)
2
dx
701 Evaluate
_
2

4
(1 + cos x){1 tan
2 x
2
tan(x + sin x) tan(x sin x)}
tan(x + sin x)
dx
http://www.artofproblemsolving.com/
This le was downloaded from the AoPS Math Olympiad Resources Page Page 4

Japan
Todays Calculation Of Integral
2011
702 f(x) is a continuous function dened in x > 0. For all a, b (a > 0, b > 0), if
_
b
a
f(x) dx is
determined by only
b
a
, then prove that f(x) =
c
x
(c : constant).
703 Given a line segment PQ with endpoints on the parabola y = x
2
such that the area bounded
by PQ and the parabola always equal to
4
3
. Find the equation of the locus of the midpoint
M.
704 A function f
n
(x) (n = 0, 1, 2, 3, ) satises the following conditions:
(i) f
0
(x) = e
2x
+ 1.
(ii) f
n
(x) =
_
x
0
(n + 2t)f
n1
(t)dt
2x
n+1
n+1
(n = 1, 2, 3, ).
Find

n=1
f

n
_
1
2
_
.
705 The parametric equations of a curve are given by x = 2(1+cos t) cos t, y = 2(1+cos t) sin t (0
t 2).
(1) Find the maximum and minimum values of x.
(2) Find the volume of the solid enclosed by the gure of revolution about the x-axis.
706 In the xyz space, consider a right circular cylinder with radius of base 2, altitude 4 such that
_
x
2
+y
2
4
0 z 4
Let V be the solid formed by the points (x, y, z) in the circular cylinder satisfying
_
z (x 2)
2
z y
2
Find the volume of the solid V .
707 In the xyz space, consider a right circular cylinder with radius of base 2, altitude 4 such that
_
x
2
+y
2
4
0 z 4
Let V be the solid formed by the points (x, y, z) in the circular cylinder satisfying
_
z (x 2)
2
z y
2
Find the volume of the solid V .
708 Find lim
n
_
1
0
x
2
| sin nx| dx (n = 1, 2, ).
http://www.artofproblemsolving.com/
This le was downloaded from the AoPS Math Olympiad Resources Page Page 5

Japan
Todays Calculation Of Integral
2011
709 Evaluate
_
1
0
x
1+x

1 x
2
dx.
710 Evaluate
_

4
0
sin (sin cos +2)
cos
4

d.
711 Evaluate
_
e
2
e
4(ln x)
2
+1
(ln x)
3
2
dx.
712 Evaluate
_

3

4
_
1
tan x (ln sin x)
+
tan x
ln cos x
_
dx.
713 If a positive sequence {a
n
}
n1
satises
_
an
0
x
n
dx = 2, then nd lim
n
a
n
.
714 Find the area enclosed by the graph of a
2
x
4
= b
2
x
2
y
2
(a > 0, b > 0).
715 Find the dierentiable function f(x) with f(0) = 0 satisfying f(x+y) = f(x)f

(y)+f

(x)f(y)
for all real numbers x, y.
716 Prove that :
_

e
1
(ln x)
n
dx = (1)
n1
n! +

e
n

m=0
(1)
nm
n!
m!
_
1
2
_
m
717 Let a
n
be the area of the part enclosed by the curve y = x
n
(n 1), the line x =
1
2
and the
x axis. Prove that :
0 ln 2
1
2
(a
1
+a
2
+ +a
n
)
1
2
n+1
718 Find

n=1
1
2
n
_
1
1
(1 x)
2
(1 +x)
n
dx (n 1).
719 Compute
_
x
0
sin t cos t sin(2 cos t) dt.
720 Evaluate
_
2
0
|x
2

2
sin
2
x| dx.
721 For constant a, nd the dierentiable function f(x) satisfying
_
x
0
(e
x
ae
t
)f(t)dt = 0.
722 Find the continuous function f(x) such that :
_
x
0
f(t)
__
t
0
f(t)dt
_
dt = f(x) +
1
2
723 Evaluate
_
e
1
{1(x1)e
x
} ln x
(1+e
x
)
2
dx.
724 Find lim
n
_
(1 +n)
1
n
_
1 +
n
2
_ 2
n
_
1 +
n
3
_ 3
n
2
_
1
n
.
http://www.artofproblemsolving.com/
This le was downloaded from the AoPS Math Olympiad Resources Page Page 6

Japan
Todays Calculation Of Integral
2011
725 For a > 1, evaluate
_
a
1
a
1
x
(ln x) ln (x
2
+ 1)dx.
726 Let P(x, y) (x > 0, y > 0) be a point on the curve C : x
2
y
2
= 1. If x =
e
u
+e
u
2
(u 0),
then nd the area bounded by the line OP, the x axis and the curve C in terms of u.
727 For positive constant a, let C : y =
a
2
(e
x
a
+ e

x
a
). Denote by l(t) the length of the part
a y t for C and denote by S(t) the area of the part bounded by the line y = t (a < t)
and C. Find lim
t
S(t)
l(t) ln t
.
728 Evaluate
_
6

12
sin x cos x x(sin x + cos x) + 1
x
2
x(sin x + cos x) + sin xcos x
dx.
729 Evaluate
_
e
1
ln x1
x
2
(ln x)
2
dx.
730 Let a
n
be the local maximum of f
n
(x) =
x
n
e
x+n
n!
(n = 1, 2, ) for x > 0.
Find lim
n
ln
_
a
2n
an
_1
n
.
731 Let C be the point of intersection of the tangent lines l, m at A(a, a
2
), B(b, b
2
) (a < b) on
the parabola y = x
2
respectively. When C moves on the parabola y =
1
2
x
2
x 2, nd the
minimum area bounded by 2 lines l, m and the parabola y = x
2
.
732 Let a be parameter such that 0 < a < 2. For 0 < x < 2, nd the extremum of F(x) =
_
x+a
x

1 cos d.
733 Find lim
n
_
1
0
x
2
e
(
x
n
)
2
dx.
734 Find the extremum of f(t) =
_
t
1
ln x
x+t
dx (t > 0).
735 Evaluate the following denite integrals:
(a)
_

2
0
xtan(x
2
) dx
(b)
_
1
3
0
xe
3x
dx
(c)
_
e
e
e
1
xln x
dx
(d)
_
3
2
x
2
+1
x(x+1)
dx
736 Evaluate
_
1
0
(e
x
+ 1){e
x
+ 1 + (1 +x +e
x
) ln(1 +x +e
x
)}
1 +x +e
x
dx
http://www.artofproblemsolving.com/
This le was downloaded from the AoPS Math Olympiad Resources Page Page 7

Japan
Todays Calculation Of Integral
2011
737 Let a, b real numbers such that a > 1, b > 1. Prove the following inequality.
_
1
1
_
1 +b
|x|
1 +a
x
+
1 +a
|x|
1 +b
x
_
dx < a +b + 2
738 Answer the following questions:
(1) Find the value of a for which S =
_

(xa sin 3x)


2
dx is minimized, then nd the minimum
value.
(2) Find the vlues of p, q for which T =
_

(sin 3x px qx
2
)
2
dx is minimized, then nd
the minimum value.
739 Find the function f(x) such that :
f(x) = cos x +
_
2
0
f(y) sin(x y) dy
740 Let r be a positive constant. If 2 curves C
1
: y =
2x
2
x
2
+1
, C
2
: y =

r
2
x
2
have each tangent
line at their point of intersection and at which their tangent lines are perpendicular each
other, then nd the area of the gure bounded by C
1
, C
2
.
741 Evaluate
_
1
0
(x 1)
2
(cos x + 1) (2x 1) sin x
(x 1 +

sin x)
2
dx
742 Evaluate
_
1
0
1 x
2
(1 +x
2
)

1 +x
4
dx
743 Evaluate
_

2
0
ln(1 +
3

sin ) cos d.
744 Let a, b be real numbers. If
_
3
0
(ax b)
2
dx 3 holds, then nd the values of a, b such that
_
3
0
(x 3)(ax b)dx is minimized.
745 When real numbers a, b move satisfying
_

0
(a cos x+b sin x)
2
dx = 1, nd the maximum value
of
_

0
(e
x
a cos x b sin x)
2
dx.
746 Prove the following inequality.
n
n
e
n+1
n!
1
4
(n + 1)
n+1
e
n+1
.
http://www.artofproblemsolving.com/
This le was downloaded from the AoPS Math Olympiad Resources Page Page 8

Japan
Todays Calculation Of Integral
2011
747 Prove that
_
4
0
_
1 cos
x
2
_
e

x
dx 2e
2
+ 30.
748 Evaluate the following integrals.
(1)
_

0
cos mxcos nx dx (m, n = 1, 2, ).
(2)
_
3
1
_
x
1
x
_
(ln x)
2
dx.
749 Let m be a positive integer. A tangent line at the point P on the parabola C
1
: y = x
2
+m
2
intersects with the parabola C
2
: y = x
2
at the points A, B. For the point Q between A and
B on C
2
, denote by S the sum of the areas of the region bounded by the line AQ,C
2
and the
region bounded by the line QB, C
2
. When Q move between A and B on C
2
, prove that the
minimum value of S doesnt depend on how we would take P, then nd the value in terms
of m.
750 Let a
n
(n 1) be the value for which
_
2x
x
e
t
n
dt (x 0) is maximal. Find lim
n
ln a
n
.
751 Find lim
n
_
1
n
_
n
0
(sin
2
x) ln(x +n)dx
1
2
ln n
_
.
752 Find f
n
(x) such that f
1
(x) = x, f
n
(x) =
_
x
0
tf
n1
(x t)dt (n = 2, 3, ).
753 Find lim
n

2n
k=1
n
2n
2
+3nk+k
2
.
754 Let S
n
be the area of the gure enclosed by a curve y = x
2
(1x)
n
(0 x 1)and the x-axis.
Find lim
n

n
k=1
S
k
.
755 Given mobile points P(0, sin ), Q(8 cos , 0)
_
0

2
_
on the x-y plane. Denote by D
the part in which line segment PQ sweeps. Find the volume V generated by a rotation of D
around the x-axis.
756 Let a be real number. A circle C touches the line y = x at the point (a, a) and passes
through the point (0, 1). Denote by P the center of C. When a moves, nd the area of the
gure enclosed by the locus of P and the line y = 1.
757 Evaluate
_
1
0
(x
2
+x + 1)
3
{ln(x
2
+x + 1) + 2}
(x
2
+x + 1)
3
(2x + 1)e
x
2
+x+1
dx.
758 Find the slope of a line passing through the point (0, 1) with which the area of the part
bounded by the line and the parabola y = x
2
is
5

5
6
.
759 Given a regular tetrahedron PQRS with side length d. Find the volume of the solid generated
by a rotation around the line passing through P and the midpoint M of QR.
http://www.artofproblemsolving.com/
This le was downloaded from the AoPS Math Olympiad Resources Page Page 9

Japan
Todays Calculation Of Integral
2011
760 Prove that there exists a positive integer n such that
_
1
0
xsin (x
2
x + 1)dx
n
n+1
sin
n+2
n+3
.
761 Find lim
n
1
n
n
_
(4n)!
(3n)!
.
762 Dene a function f
n
(x) (n = 0, 1, 2, ) by
f
0
(x) = sin x, f
n+1
(x) =
_
2
0
f
n
(t) sin(x +t)dt.
(1) Let f
n
(x) = a
n
sin x +b
n
cos x. Express a
n+1
, b
n+1
in terms of a
n
, b
n
.
(2) Find

n=0
f
n
_

4
_
.
763 Evaluate
_
4
1
x2
(x
2
+4)

x
dx.
764 Let f(x) be a continuous function dened on 0 x and satises f(0) = 1 and
__

0
(sin x + cos x)f(x)dx
_
2
=
_

0
{f(x)}
2
dx.
Evaluate
_

0
{f(x)}
3
dx.
765 Dene two functions g(x), f(x) (x 0) by g(x) =
_
x
0
e
t
2
dt, f(x) =
_
1
0
e
(1+s
2
)x
1+s
2
ds.
Now we know that f

(x) =
_
1
0
e
(1+s
2
)x
ds.
(1) Find f(0).
(2) Show that f(x)

4
e
x
(x 0).
(3) Let h(x) = {g(

x)}
2
. Show that f

(x) = h

(x).
(4) Find lim
x+
g(x)
Please solve the problem without using Double Integral or Jacobian for those Japanese High
School Students who dont study them.
766 Let f(x) be a continuous function dened on 0 x and satises f(0) = 1 and
__

0
(sin x + cos x)f(x)dx
_
2
=
_

0
{f(x)}
2
dx.
Evaluate
_

0
{f(x)}
3
dx.
767 For 0 t 1, dene f(t) =
_
2
0
| sin x t|dx. Evaluate
_
1
0
f(t)dt.
http://www.artofproblemsolving.com/
This le was downloaded from the AoPS Math Olympiad Resources Page Page 10

Japan
Todays Calculation Of Integral
2011
768 Let r be a real such that 0 < r 1. Denote by V (r) the volume of the solid formed by all
points of (x, y, z) satisfying
x
2
+y
2
+z
2
1, x
2
+y
2
r
2
in xyz-space.
(1) Find V (r).
(2) Find lim
r10
V (1)V (r)
(1r)
3
2
.
(3) Find lim
r+0
V (r)
r
2
.
769 In xyz space, nd the volume of the solid expressed by x
2
+y
2
z

3y + 1.
http://www.artofproblemsolving.com/
This le was downloaded from the AoPS Math Olympiad Resources Page Page 11

Japan
Todays Calculation Of Integral
2012
770 Find the value of a such that :
101a = 6539
_
1
1
x
12
+ 31
1 + 2011
x
dx.
771 (1) Find the range of a for which there exist two common tangent lines of the curve y =
8
27
x
3
and the parabola y = (x +a)
2
other than the x axis.
(2) For the range of a found in the previous question, express the area bounded by the two
tangent lines and the parabola y = (x +a)
2
in terms of a.
772 Given are three points A(2, 0, 2), B(1, 1, 0), C(0, 0, 3) in the coordinate space. Find the
volume of the solid of a triangle ABC generated by a rotation about z-axis.
773 For x 0 nd the value of x by which f(x) =
_
x
0
3
t
(3
t
4)(x t)dt is minimized.
774 Find the real number a such that
_
a
0
e
x
+e
x
2
dx =
12
5
.
775 Let a be negative constant. Find the value of a and f(x) such that
_
t
2
a
2
f(x)dx = t
2
+ 3t 4
holds for any real numbers t.
776 Evaluate
_
1+

5
2
1

5
2
(2x
2
1)e
2x
dx.
777 Given two points P, Q on the parabola C : y = x
2
x 2 in the xy plane. Note that the x
coodinate of P is less than that of Q.
(a) If the origin O is the midpoint of the linesegment PQ, then nd the equation of the line
PQ.
(b) If the origin O divides internally the line segment PQ by 2:1, then nd the equation of
PQ.
(c) If the origin O divides internally the line segment PQ by 2:1, nd the area of the gure
bounded by the parabola C and the line PQ.
778 In the xyz space with the origin O, Let K
1
be the surface and inner part of the sphere
centered on the point (1, 0, 0) with radius 2 and let K
2
be the surface and inner part of the
sphere centered on the point (1, 0, 0) with radius 2. For three points P, Q, R in the space,
consider points X, Y dened by

OX =

OP +

OQ,

OY =
1
3
(

OP +

OQ+

OR).
(1) When P, Q move every cranny in K
1
, K
2
respectively, nd the volume of the solid
generated by the whole points of the point X.
http://www.artofproblemsolving.com/
This le was downloaded from the AoPS Math Olympiad Resources Page Page 1

Japan
Todays Calculation Of Integral
2012
(2) Find the volume of the solid generated by the whole points of the point R for which for
any P belonging to K
1
and any Q belonging to K
2
, Y belongs to K
1
.
(3) Find the volume of the solid generated by the whole points of the point R for which for
any P belonging to K
1
and any Q belonging to K
2
, Y belongs to K
1
K
2
.
779 Consider parabolas C
a
: y = 2x
2
+ 4ax 2a
2
+a + 1 and C : y = x
2
2x in the coordinate
plane. When C
a
and C have two intersection points, nd the maximum area enclosed by
these parabolas.
780 Let n 3 be integer. Given a regular n-polygon P with side length 4 on the plane z = 0 in
the xyz-space.Llet G be a circumcenter of P. When the center of the sphere B with radius
1 travels round along the sides of P, denote by K
n
the solid swept by B.
Answer the following questions.
(1) Take two adjacent vertices P
1
, P
2
of P. Let Q be the intersection point between the
perpendicular dawn from G to P
1
P
2
, prove that GQ > 1.
(2) (i) Express the area of cross section S(t) in terms of t, n when K
n
is cut by the plane
z = t (1 t 1).
(ii) Express the volume V (n) of K
n
in terms of n.
(3) Denote by l the line which passes through G and perpendicular to the plane z = 0.
Express the volume W(n) of the solid by generated by a rotation of K
n
around l in terms of
n.
(4) Find lim
n
V (n)
W(n)
.
781 Let l, m be the tangent lines passing through the point A(a, a1) on the line y = x1 and
touch the parabola y = x
2
. Note that the slope of l is greater than that of m.
(1) Exress the slope of l in terms of a.
(2) Denote P, Q be the points of tangency of the lines l, m and the parabola y = x
2
. Find
the minimum area of the part bounded by the line segment PQ and the parabola y = x
2
.
(3) Find the minimum distance between the parabola y = x
2
and the line y = x 1.
782 Let C be the part of the graph y =
1
x
(x > 0). Take a point P
_
t,
1
t
_
(t > 0) on C.
(i) Find the equation of the tangent l at the point A(1, 1) on the curve C.
(ii) Let m be the line passing through the point P and parallel to l. Denote Q be the
intersection point of the line m and the curve C other than P. Find the coordinate of Q.
(iii) Express the area S of the part bounded by two line segments OP, OQ and the curve C
for the origin O in terms of t.
http://www.artofproblemsolving.com/
This le was downloaded from the AoPS Math Olympiad Resources Page Page 2

Japan
Todays Calculation Of Integral
2012
(iv) Express the volume V of the solid generated by a rotation of the part enclosed by two
lines passing through the point P and pararell to the y-axis and passing through the point Q
and pararell to y-axis, the curve C and the x-axis in terms of t.
(v) lim
t10
S
V
.
783 Dene a sequence a
1
= 0,
1
1a
n+1

1
1an
= 2n + 1 (n = 1, 2, 3, ).
(1) Find a
n
.
(2) Let b
k
=
_
k+1
k
(1

a
k+1
) for k = 1, 2, 3, .
Prove that

n
k=1
b
k
<

2 1 for each n.
Last Edited
784 Dene for positive integer n, a function f
n
(x) =
ln x
x
n
(x > 0). In the coordinate plane, denote
by S
n
the area of the gure enclosed by y = f
n
(x) (x t), the x-axis and the line x = t
and denote by T
n
the area of the rectagle with four vertices (1, 0), (t, 0), (t, f
n
(t)) and
(1, f
n
(t)).
(1) Find the local maximum f
n
(x).
(2) When t moves in the range of t > 1, nd the value of t for which T
n
(t)S
n
(t) is maximized.
(3) Find S
1
(t) and S
n
(t) (n 2).
(4) For each n 2, prove that there exists the only t > 1 such that T
n
(t) = S
n
(t).
Note that you may use lim
x
ln x
x
= 0.
785 For a positive real number x, nd the minimum value of f(x) =
_
2x
x
(t ln t t)dt.
786 For each positive integer n, dene H
n
(x) = (1)
n
e
x
2
d
n
dx
n
e
x
2
.
(1) Find H
1
(x), H
2
(x), H
3
(x).
(2) Express
d
dx
H
n
(x) interms of H
n
(x), H
n+1
(x). Then prove that H
n
(x) is a polynpmial
with degree n by induction.
(3) Let a be real number. For n 3, express S
n
(a) =
_
a
0
xH
n
(x)e
x
2
dx in terms of
H
n1
(a), H
n2
(a), H
n2
(0).
(4) Find lim
a
S
6
(a). If necessary, you may use lim
x
x
k
e
x
2
= 0 for a positive integer
k.
787 Take two points A (1, 0), B (1, 0) on the xy-plane. Let F be the gure by which the whole
points P on the plane satises

4
APB and the gure formed by A, B.
Answer the following questions:
http://www.artofproblemsolving.com/
This le was downloaded from the AoPS Math Olympiad Resources Page Page 3

Japan
Todays Calculation Of Integral
2012
(1) Illustrate F.
(2) Find the volume of the solid generated by a rotation of F around the x-axis.
788 For a function f(x) = ln(1 +

1 x
2
)

1 x
2
ln x (0 < x < 1), answer the following
questions:
(1) Find f

(x).
(2) Sketch the graph of y = f(x).
(3) Let P be a mobile point on the curve y = f(x) and Q be a point which is on the tangent
at P on the curve y = f(x) and such that PQ = 1. Note that the x-coordinate of Q is les
than that of P. Find the locus of Q.
789 Find the non-constant function f(x) such that f(x) = x
2

_
1
0
(f(t) +x)
2
dt.
790 Dene a parabola C by y = x
2
+ 1 on the coordinate plane. Let s, t be real numbers with
t < 0. Denote by l
1
, l
2
the tangent lines drawn from the point (s, t) to the parabola C.
(1) Find the equations of the tangents l
1
, l
2
.
(2) Let a be positive real number. Find the pairs of (s, t) such that the area of the region
enclosed by C, l
1
, l
2
is a.
791 Let S be the domain in the coordinate plane determined by two inequalities:
y
1
2
x
2
,
x
2
4
+ 4y
2

1
8
.
Denote by V
1
the volume of the solid by a rotation of S about the x-axis and by V
2
, by a
rotation of S about the y-axis.
(1) Find the values of V
1
, V
2
.
(2) Compare the size of the value of
V
2
V
1
and 1.
792 Answer the following questions:
(1) Let a be positive real number. Find lim
n
(1 +a
n
)
1
n
.
(2) Evaluate
_

3
1
1
x
2
ln

1 +x
2
dx.
35 points
793 Find the area of the gure bounded by two curves y = x
4
, y = x
2
+ 2.
794 Dene a function f(x) =
_

2
0
cos |tx|
1+sin |tx|
dt for 0 x .
Find the maximum and minimum value of f(x) in 0 x .
http://www.artofproblemsolving.com/
This le was downloaded from the AoPS Math Olympiad Resources Page Page 4

Japan
Todays Calculation Of Integral
2012
795 Evaluate
_

2

3
2+sin x
1+cos x
dx.
796 Answer the following questions:
(1) Let a be non-zero constant. Find
_
x
2
cos(a ln x)dx.
(2) Find the volume of the solid generated by a rotation of the gures enclosed by the curve
y = xcos(ln x), the x-axis and the lines x = 1, x = e

4
about the x-axis.
797 In the xyz-space take four points P(0, 0, 2), A(0, 2, 0), B(

3, 1, 0), C(

3, 1, 0).
Find the volume of the part satifying x
2
+y
2
1 in the tetrahedron PABC.
50 points
798 Denote by C, l the graphs of the cubic function C : y = x
3
3x
2
+ 2x, the line l : y = ax.
(1) Find the range of a such that C and l have intersection point other than the origin.
(2) Denote S(a) by the area bounded by C and l. If a move in the range found in (1), then
nd the value of a for which S(a) is minimized.
50 points
799 Let n be positive integer. Dene a sequence {a
k
} by
a
1
=
1
n(n + 1)
, a
k+1
=
1
k +n + 1
+
n
k
k

i=1
a
i
(k = 1, 2, 3, ).
(1) Find a
2
and a
3
.
(2) Find the general term a
k
.
(3) Let b
n
=

n
k=1

a
k
. Prove that lim
n
b
n
= ln 2.
50 points
800 For a positive constant a, nd the minimum value of f(x) =
_

2
0
| sin t axcos t|dt.
801 Answer the following questions:
(1) Let f(x) be a function such that f

(x) is continuous and f

(a) = f

(b) = 0 for some a < b.


Prove that f(b) f(a) =
_
b
a
_
a+b
2
x
_
f

(x)dx.
(2) Consider the running a car on straight road. After a car which is at standstill at a
trac light started at time 0, it stopped again at the next trac light apart a distance L at
time T. During the period, prove that there is an instantfor which the absolute value of the
acceleration of the car is more than or equal to
4L
T
2
.
http://www.artofproblemsolving.com/
This le was downloaded from the AoPS Math Olympiad Resources Page Page 5

Japan
Todays Calculation Of Integral
2012
802 Let k and a are positive constants. Denote by V
1
the volume of the solid generated by a
rotation of the gure enclosed by the curve C : y =
x
x+k
(x 0), the line x = a and the x-axis
around the x-axis, and denote by V
2
that of the solid by a rotation of the gure enclosed by
the curve C, the line y =
a
a+k
and the y-axis around the y-axis. Find the ratio
V
2
V
1
.
803 Answer the following questions:
(1) Evaluate
_
1
1
(1 x
2
)e
2x
dx.
(2) Find lim
n
_
(2n)!
n!n
n
_1
n
.
804 For a > 0, nd the minimum value of I(a) =
_
e
1
| ln ax| dx.
805 Prove the following inequalities:
(1) For 0 x 1,
1
1
3
x
1

1 +x
2
1.
(2)

3

1
6

_

3
2
0
1

1x
4
dx

3
.
806 Let n be positive integers and t be a positive real number. Evaluate
_
2n
t

0
|xsin tx| dx.
807 Dene a sequence a
n
satisfying :
a
1
= 1, a
n+1
=
na
n
2 +n(a
n
+ 1)
(n = 1, 2, 3, ).
Find lim
m
m

2m
n=m+1
a
n
.
808 For a constant c, a sequence a
n
is dened by a
n
=
_
1
c
nx
n1
_
ln
_
1
x
__
n
dx (n = 1, 2, 3, ).
Find lim
n
a
n
.
809 For a > 0, denote by S(a) the area of the part bounded by the parabolas y =
1
2
x
2
3a and
y =
1
2
x
2
+ 2ax a
3
a
2
. Find the maximum area of S(a).
810 Given the functions f(x) = xe
x
+2x
_
2
0
|g(t)|dt1, g(x) = x
2
x
_
1
0
f(t)dt, evaluate
_
2
0
|g(t)|dt.
811 Let a be real number. Evaluate
_
a+
a
|x| cos x dx.
812 Let f(x) =
cos 2x(a+2) cos x+a+1
sin x
. For constant a such that lim
x0
f(x)
x
=
1
2
, evaluate
_

2

3
1
f(x)
dx.
http://www.artofproblemsolving.com/
This le was downloaded from the AoPS Math Olympiad Resources Page Page 6

Japan
Todays Calculation Of Integral
2012
813 Let a be a real number. Find the minimum value of
_
1
0
|ax x
3
|dx.
How many solutions (including University Mathematics )are there for the problem?
Any advice would be appreciated. ! s:) img src=SMILIES
P
ATH/smile.gifalt = :
)title = Smile/ ><! s :) > FindtheareaoftheregionboundedbyC: y=-x
4
+ 8x
3

18x
2
+ 11 and the tangent line which touches C at distinct two points.
814 815 Prove that :

n
i=0
_
1 sin
i
4n
cos
i
4n
_

< 1.
816 Find the volume of the solid of a circle x
2
+ (y 1)
2
= 4 generated by a rotation about the
x-axis.
817 Dene two functions f(t) =
1
2
_
t +
1
t
_
, g(t) = t
2
2 ln t. When real number t moves in the
range of t > 0, denote by C the curve by which the point (f(t), g(t)) draws on the xy-plane.
Let a > 1, nd the area of the part bounded by the line x =
1
2
_
a +
1
a
_
and the curve C.
818 For a function f(x) = x
3
x
2
+x, nd the limit lim
n
_
2n
n
1
f
1
(x)
3
+|f
1
(x)|
dx.
819 For real numbers a, b with 0 a , a < b, let I(a, b) =
_
b
a
e
x
sin x dx.
Determine the value of a such that lim
b
I(a, b) = 0.
820 Let P
k
be a point whose x-coordinate is 1 +
k
n
(k = 1, 2, , n) on the curve y = ln x. For
A(1, 0), nd the limit lim
n
1
n

n
k=1
AP
k
2
.
821 Prove that : ln
11
27
<
_
3
4
1
4
1
ln(1x)
dx < ln
7
15
.
822 For n = 0, 1, 2, , let
a
n
=
_
n+1
n
{xe
x
(n + 1)e
n1
(x n)} dx,
b
n
=
_
n+1
n
{xe
x
(n + 1)e
n1
} dx.
Find lim
n

n
k=0
(a
k
b
k
).
823 Let C be the curve expressed by x = sin t, y = sin 2t
_
0 t

2
_
.
(1) Express y in terms of x.
(2) Find the area of the gure D enclosed by the x-axis and C.
(3) Find the volume of the solid generated by a rotation of D about the y-axis.
824 In the xy-plane, for a > 1 denote by S(a) the area of the gure bounded by the curve
y = (a x) ln x and the x-axis.
Find the value of integer n for which lim
a
S(a)
a
n
ln a
is non-zero real number.
http://www.artofproblemsolving.com/
This le was downloaded from the AoPS Math Olympiad Resources Page Page 7

Japan
Todays Calculation Of Integral
2012
825 Answer the following questions.
(1) For x 0, show that x
x
3
6
sin x x.
(2) For x 0, show that
x
3
3

x
5
30

_
x
0
t sin t dt
x
3
3
.
(3) Find the limit
lim
x0
sin x xcos x
x
3
.
826 Let G be a hyper elementary abelian pgroup and let f : G G be a homomorphism. Then
prove that ker f is isomorphic to cokerf.
827 Find lim
n

k=0
_
(2k+1)
2k
xe
x
sin x dx.
828 Find a function f(x), which is dierentiable and f

(x) is continuous, such that


_
x
0
f(t) cos(x
t) dt = xe
2x
.
829 Let a be a positive constant. Find the value of ln a such that
_
e
1
ln(ax) dx
_
e
1
x dx
=
_
e
1
ln(ax)
x
dx.
830 Find lim
n
1
(ln n)
2

n
k=3
ln k
k
.
831 Let n be a positive integer. Answer the following questions.
(1) Find the maximum value of f
n
(x) = x
n
e
x
for x 0.
(2) Show that lim
x
f
n
(x) = 0.
(3) Let I
n
=
_
x
0
f
n
(t) dt. Find lim
x
I
n
(x).
832 Find the limit
lim
n
1
nln n
_
(n+1)

(sin
2
t)(ln t) dt.
833 Let f(x) =
_
x
0
e
t
(cos t + sin t) dt, g(x) =
_
x
0
e
t
(cos t sin t) dt.
For a real number a, nd

n=1
e
2a
{f
(n)
(a)}
2
+{g
(n)
(a)}
2
.
834 Find the maximum and minimum areas of the region enclosed by the curve y = |x|e
|x|
and
the line y = a (0 a e) at [1, 1].
http://www.artofproblemsolving.com/
This le was downloaded from the AoPS Math Olympiad Resources Page Page 8

Japan
Todays Calculation Of Integral
2012
835 Evaluate the following denite integrals.
(a)
_
2
1
x1
x
2
2x+2
dx
(b)
_
1
0
e
4x
e
2x
+2
dx
(c)
_
e
1
xln

x dx
(d)
_

3
0
_
cos
2
xsin 3x
1
4
sin 5x
_
dx
836 Evaluate
_

0
e
sin x
cos
2
(sin x) cos x dx.
837 Let f
n
(x) =

n
k=1
(1)
k+1
_
x
2k1
2k1
+
x
2k
2k
_
.
Find lim
n
f
n
(1).
838 Prove that :
e1
e
<
_
1
0
e
x
2
dx <

4
.
839 Evaluate
_
1
1
2

1 x
2
dx.
840 Let x, y be real numbers. For a function f(t) = xsin t + y cos t, draw the domain of the
points (x, y) for which the following inequality holds.

f(t) cos t dt

{f(t)}
2
dt.
841 Find
_
x
0
dt
1+t
2
+
_
1
x
0
dt
1+t
2
(x > 0).
842 Let S
n
=
_

0
sin
n
x dx (n = 1, 2, , ). Find lim
n
nS
n
S
n+1
.
843 Let f(x) be a continuous function such that
_
1
0
f(x) dx = 1. Find f(x) for which
_
1
0
(x
2
+x+
1)f(x)
2
dx is minimized.
844 Let be a solution satisfying the equation |x| = e
x
. Let I
n
=
_

0
(xe
nx
+ x
n1
)dx (n =
1, 2, ). Find lim
n
n
2
I
n
.
845 Consider for a real number t > 1, I(t) =
_
4t4
4
(x 4)

x + 4 dx.
Find the minimum value of I(t) (t > 1).
846 For a > 0, let f(a) = lim
t0
_
1
t
|ax + xln x| dx. Let a vary in the range 0 < a < +, nd
the minimum value of f(a).
http://www.artofproblemsolving.com/
This le was downloaded from the AoPS Math Olympiad Resources Page Page 9

Japan
Todays Calculation Of Integral
2012
847 Consider a right-angled triangle with AB = 1, AC =

3, BAC =

2
. Let P
1
, P
2
, , P
n1
(n
2) be the points which are closest from A, in this order and obtained by dividing n equally
parts of the line segment AB. Denote by A = P
0
, B = P
n
, answer the questions as below.
(1) Find the inradius of P
k
CP
k+1
(0 k n 1).
(2) Denote by S
n
the total sum of the area of the incircle for P
k
CP
k+1
(0 k n 1).
Let I
n
=
1
n

n1
k=0
1
3+(
k
n
)
2
, show that nS
n

3
4
I
n
, then nd the limit lim
n
I
n
.
(3) Find the limit lim
n
nS
n
.
848 Evaluate
_

4
0
sin 2 ln
1sin
cos
(1+cos 2)

ln
1+sin
cos
d.
849 Evaluate
_
e
2
1
(2x
2
+2x+1)e
x

x
dx.
850 Evaluate
_

0
{(1 xsin 2x)e
cos
2
x
+ (1 +xsin 2x)e
sin
2
x
} dx.
851 Let T be a period of a function f(x) = | cos x| sin x (, ). Find lim
n
_
nT
0
e
x
f(x) dx.
852 Let f(x) be a polynomial. Prove that if
_
1
0
f(x)g
n
(x) dx = 0 (n = 0, 1, 2, ), then all
coecients of f(x) are 0 for each case as follows.
(1) g
n
(x) = (1 +x)
n
(2) g
n
(x) = sin nx
(3) g
n
(x) = e
nx
853 Let 0 < a <

2
. Find lim
a+0
1
a
3
_
a
0
ln (1 + tan a tan x) dx.
854 Given a gure F : x
2
+
y
2
3
= 1 on the coordinate plane. Denote by S
n
the area of the
common part of the n+1

s gures formed by rotating F of


k
2n
(k = 0, 1, 2, , n) radians
counterclockwise about the origin. Find lim
n
S
n
.
855 Let f(x) be a function which is dierentiable twice and f

(x) > 0 on [0, 1].


For a positive integer n, nd lim
n
n
_
_
1
0
f(x) dx
1
n

n1
k=0
f
_
k
n
_
_
.
856 On the coordinate plane, nd the area of the part enclosed by the curve C : (a + x)y
2
=
(a x)x
2
(x 0) for a > 0.
http://www.artofproblemsolving.com/
This le was downloaded from the AoPS Math Olympiad Resources Page Page 10

Japan
Todays Calculation Of Integral
2012
857 Let f(x) = lim
n
(cos
n
x + sin
n
x)
1
n
for 0 x

2
.
(1) Find f(x).
(2) Find the volume of the solid generated by a rotation of the gure bounded by the curve
y = f(x) and the line y = 1 around the y-axis.
858 On the plane S in a space, given are unit circle C with radius 1 and the line L. Find
the volume of the solid bounded by the curved surface formed by the point P satifying the
following condition (a), (b).
(a) The point of intersection Q of the line passing through P and perpendicular to S are on
the perimeter or the inside of C.
(b) If A, B are the points of intersection of the line passing through Q and pararell to L,
then PQ = AQ BQ.
859 In the x-y plane, for t > 0, denote by S(t) the area of the part enclosed by the curve y = e
t
2
x
,
the x-axis, y-axis and the line x =
1
t
. Show that S(t) >
4
3
. If necessary, you may use e
3
> 20.
http://www.artofproblemsolving.com/
This le was downloaded from the AoPS Math Olympiad Resources Page Page 11

Japan
Todays Calculation Of Integral
2013
860 For a function f(x) (x 1) satisfying f(x) = (log
e
x)
2

_
e
1
f(t)
t
dt, answer the questions as
below.
(a) Find f(x) and the y-coordinate of the inection point of the curve y = f(x).
(b) Find the area of the gure bounded by the tangent line of y = f(x) at the point (e, f(e)),
the curve y = f(x) and the line x = 1.
861 Answer the questions as below.
(1) Find the local minimum of y = x(1 x
2
)e
x
2
.
(2) Find the total area of the part bounded the graph of the function in (1) and the x-axis.
862 Draw a tangent with positive slope to a parabola y = x
2
+1. Find the x-coordinate such that
the area of the gure bounded by the parabola, the tangent and the coordinate axisis is
11
3
.
863 For 0 < t 1, let F(t) =
1
t
_

2
t
0
| cos 2x| dx.
(1) Find lim
t0
F(t).
(2) Find the range of t such that F(t) 1.
864 Let m, n be positive integer such that 2 m < n.
(1) Prove the inequality as follows.
n + 1 m
m(n + 1)
<
1
m
2
+
1
(m+ 1)
2
+ +
1
(n 1)
2
+
1
n
2
<
n + 1 m
n(m 1)
(2) Prove the inequality as follows.
3
2
lim
n
_
1 +
1
2
2
+ +
1
n
2
_
2
(3) Prove the inequality which is made precisely in comparison with the inequality in (2) as
follows.
29
18
lim
n
_
1 +
1
2
2
+ +
1
n
2
_

61
36
865 Find the volume of the solid generated by a rotation of the region enclosed by the curve
y = x
3
x and the line y = x about the line y = x as the axis of rotation.
http://www.artofproblemsolving.com/
This le was downloaded from the AoPS Math Olympiad Resources Page Page 1

Japan
Todays Calculation Of Integral
2013
866 Given a solid R contained in a semi cylinder with the hight 1 which has a semicircle with
radius 1 as the base. The cross section at the hight x (0 x 1) is the form combined with
two right-angled triangles as attached gure as below. Answer the following questions.
(1) Find the cross-sectional area S(x) at the hight x.
(2) Find the volume of R. If necessary, when you integrate, set x = sin t.
867 Express
_
2
0
f(x)dx for any quadratic functions f(x) in terms of f(0), f(1) and f(2).
868 In the coordinate space, dene a square S, dened by the inequality |x| 1, |y| 1 on the
xy-plane, with four vertices A(1, 1, 0), B(1, 1, 0), C(1, 1, 0), D(1, 1, 0). Let V
1
be
the solid by a rotation of the square S about the line BD as the axis of rotation, and let V
2
be the solid by a rotation of the square S about the line AC as the axis of rotation.
(1) For a real number t such that 0 t < 1, nd the area of cross section of V
1
cut by the
plane x = t.
(2) Find the volume of the common part of V
1
and V
2
.
869 Let I
n
=
1
n+1
_

0
x(sin nx +n cos nx)dx (n = 1, 2, ).
Answer the questions below.
(1) Find I
n
.
(2) Find

n=1
I
n
.
870 Consider the ellipse E : 3x
2
+y
2
= 3 and the hyperbola H : xy =
3
4
.
(1) Find all points of intersection of E and H.
(2) Find the area of the region expressed by the system of inequality
_
3x
2
+y
2
3
xy
3
4
,
871 Dene sequences {a
n
}, {b
n
} by
a
n
=
_
6

6
e
nsin
d, b
n
=
_
6

6
e
nsin
cos d (n = 1, 2, 3, ).
(1) Find b
n
.
(2) Prove that for each n, b
n
a
n

2

3
b
n
.
(3) Find lim
n
1
n
ln(na
n
).
http://www.artofproblemsolving.com/
This le was downloaded from the AoPS Math Olympiad Resources Page Page 2

Japan
Todays Calculation Of Integral
2013
872 Let n be a positive integer.
(1) For a positive integer k such that 1 k n, Show that :
_ k
2n

k1
2n

sin 2nt cos t dt = (1)
k+1
2n
4n
2
1
_
cos
k
2n
+ cos
k 1
2n

_
.
(2) Find the area S
n
of the part expressed by a parameterized curve C
n
: x = sin t, y =
sin 2nt (0 t ).
If necessary, you may use

n1
k=1
cos
k
2n
=
1
2
_
1
tan

4n
1
_
(n 2).
(3) Find lim
n
S
n
.
873 Let a, b be positive real numbers. Consider the circle C
1
: (x a)
2
+y
2
= a
2
and the ellipse
C
2
: x
2
+
y
2
b
2
= 1.
(1) Find the condition for which C
1
is inscribed in C
2
.
(2) Suppose b =
1

3
and C
1
is inscribed in C
2
. Find the coordinate (p, q) of the point of
tangency in the rst quadrant for C
1
and C
2
.
(3) Under the condition in (1), nd the area of the part enclosed by C
1
, C
2
for x p.
60 point
874 Given a parabola C : y = 1 x
2
in xy-palne with the origin O. Take two points P(p, 1
p
2
), Q(q, 1 q
2
) (p < q) on C.
(1) Express the area S of the part enclosed by two segments OP, OQ and the parabalola C
in terms of p, q.
(2) If q = p + 1, then nd the minimum value of S.
(3) If pq = 1, then nd the minimum value of S.
875 Evaluate
_
1
0
x
2
+x+1
x
4
+x
3
+x
2
+x+1
dx.
876 Suppose a function f(x) is continuous on [1, 1] and satises the condition :
1) f(1) f(1).
2) x +f(x) is non decreasing function.
3)
_
1
1
f(x) dx = 0.
Show that
_
1
1
f(x)
2
dx
2
3
.
877 Let f(x) = lim
n
sin
n+2
x+cos
n+2
x
sin
n
x+cos
n
x
for 0 x

2
.
Evaluate
_

2
0
f(x) dx.
http://www.artofproblemsolving.com/
This le was downloaded from the AoPS Math Olympiad Resources Page Page 3

Japan
Todays Calculation Of Integral
2013
878 A cubic function f(x) satises the equation sin 3t = f(sin t) for all real numbers t.
Evaluate
_
1
0
f(x)
2

1 x
2
dx.
879 Evaluate the integrals as follows.
(1)
_
x
2
2x
dx
(2)
_
3

x
5
+x
3
dx
(3)
_
1
0
(1 x) cos x dx
880 For a > 2, let f(t) =
sin
2
at+t
2
at sin at
, g(t) =
sin
2
att
2
at sin at
_
0 < |t| <

2a
_
and
let C : x
2
y
2
=
4
a
2
_
x
2
a
_
. Answer the questions as follows.
(1) Show that the point (f(t), g(t)) lies on the curve C.
(2) Find the normal line of the curve C at the point (lim
t0
f(t), lim
t0
g(t)) .
(3) Let V (a) be the volume of the solid generated by a rotation of the part enclosed by the
curve C, the nornal line found in (2) and the x-axis. Express V (a) in terms of a, then nd
lim
a
V (a).
881 Evaluate
_

2013
k=1
sin kx
_
2
dx.
882 Find lim
n

n
k=1
1
n+k
(ln(n +k) ln n).
883 Prove that for each positive integer n
4n
2
+ 1
4n
2
1
_

0
(e
x
e
x
) cos 2nx dx >
e

2
4
ln
(2n + 1)
2
(2n 1)(n + 3)
.
884 Prove that :
(e 1) <
_

0
e
| cos 4x|
dx < 2(e

2
1)
885 Find the innite integrals as follows.
(1) 2013 Hiroshima City University entrance exam/Informatic Science
_
x
2
2x
2
dx
(2) 2013 Kanseigakuin University entrance exam/Science and Technology
_
x
4
ln x dx
(3) 2013 Shinsyu University entrance exam/Textile Science and Technology, Second-exam
_
cos
3
x
sin
2
x
dx
http://www.artofproblemsolving.com/
This le was downloaded from the AoPS Math Olympiad Resources Page Page 4

Japan
Todays Calculation Of Integral
2013
886 Find the functions f(x), g(x) such that
f(x) = e
x
sin x +
_

0
ug(u) du
g(x) = e
x
cos x +
_

0
uf(u) du
887 For the function f(x) =
_
x
0
dt
1+t
2
, answer the questions as follows.
Note : Please solve the problems without using directly the formula
_
1
1+x
2
dx = tan
1
x+C
for Japanese High School students those who dont study arc sin x, arc cos x, arc tanx.
(1) Find f(

3)
(2) Find
_

3
0
xf(x) dx
(3) Prove that for x > 0. f(x) +f
_
1
x
_
is constant, then nd the value.
888 In the coordinate plane, given a circle K : x
2
+y
2
= 1, C : y = x
2
2. Let l be the tangent
line of K at P(cos , sin ) ( < < 2). Find the minimum area of the part enclosed by l
and C.
889 Find the area S of the region enclosed by the curve y =

x
1
x

(x > 0) and the line y = 2.


890 A function f
n
(x) (n = 1, 2, ) is dened by f
1
(x) = x and
f
n
(x) = x +
e
2
_
1
0
f
n1
(t)e
xt
dt (n = 2, 3, )
. Find f
n
(x).
891 Given a triangle OAB with the vetices O(0, 0, 0), A(1, 0, 0), B(1, 1, 0) in the xyz space.
Let V be the cone obtained by rotating the triangle around the x-axis. Find the volume of
the solid obtained by rotating the cone V around the y-axis.
893 Find the minimum value of f(x) =
_

4
0
| tan t x|dt.
894 Let a be non zero real number. Find the area of the gure enclosed by the line y = ax, the
curve y = xln(x + 1).
895 In the coordinate plane, suppose that the parabola C : y =
p
2
x
2
+q (p > 0, q > 0) touches
the circle with radius 1 centered on the origin at distinct two points. Find the minimum area
of the gure enclosed by the part of y 0 of C and the x-axis.
896 Given sequences a
n
=
1
n
n

2n
P
n
, b
n
=
1
n
2
n

4n
P
2n
and c
n
=
n
_
8n
P
4n
6n
P
4n
, nd lim
n
a
n
, lim
n
b
n
and
lim
n
c
n
.
http://www.artofproblemsolving.com/
This le was downloaded from the AoPS Math Olympiad Resources Page Page 5

You might also like